PaEasy Emergency Medicine

Pataasin ang iyong marka sa homework at exams ngayon gamit ang Quizwiz!

A 52-year-old male has a 200-pound file cabinet fall on his right leg. He comes to the emergency department complaining of pain and swelling to the right leg. He is also complaining of parasthesias to the leg also. Based on these findings on history, what part of the leg would be the most likely site of compartment syndrome? A Anterior compartment of the leg B Deep posterior compartment of the leg C Dorsal compartment of the forearm D Lateral compartment of the leg E Superficial posterior compartment of the leg

Anterior compartment of the leg Anterior compartment syndrome is most commonly found in the anterior compartment of the leg, with the volar compartment of the forearm also a common location. The mechanism is generally an acute crushing trauma to the affected area that causes an increase in pressure within the compartment that inhibits venous outflow and a decrease in arterial blood flow. This adversely affects tissue perfusion and ischemia of the involved tissues can occur. The symptoms can also occur with chronic exertion or when there is a dramatic increase in the amount of exercise being performed. Anterior compartment syndrome of the leg is likely most common due to its vulnerable location and susceptibility to injury in athletics and motor vehicle accidents. The rate of occurrence of acute anterior compartment syndrome is also likely due to the fact that the anterior compartment lies adjacent to the tibia and the tibia is the most frequently fractured long bone. Compartment syndrome could occur in any compartment of the upper or lower extremities if the necessary circumstances were in place (either acute trauma to the area or excessive use of the muscles that are associated with each of the compartments)

A 23-year-old college basketball player twists her ankle while practicing. She explains the injury that is consistent with an inversion mechanism. Based on this history, what ligament would you expect to be the most likely injured in the ankle of this patient? A Anterior talofibular B Anterior tibiofibular C Calcaneofibular D Deltoid E Posterior talofibular ligament

Anterior talofibular Greater than 25,000 ankle sprains happen in the USA every day and the vast majority of those are inversion ankle sprains. The anterior talofibular ligament is the first, and often only, ligament damaged in inversion ankle sprains. As the force of the inversion increases, other lateral ankle ligaments can be involved. When the anterior tibiofibular ligament is involved, this is referred to as a high ankle sprain and such injuries generally have a prolonged recovery time. Calcaneofibular ligaments are generally the second most frequently injured of the lateral ankle ligaments and when injury occurs it is typically in combination with the anterior talofibular ligament. The deltoid ligament is a very strong ligament on the medial aspect of the ankle. Eversion stresses the deltoid ligament, but strong eversion forces are rare and when they do occur, an avulsion fracture of the medial malleolus is more likely than a significant ligament tear. The posterior talofibular ligament is one of the lateral ankle ligaments and can be injured in an inversion injury, but the rate of injury to this ligament lags far behind the anterior talofibular or calcaneofibular ligaments.

A 36-year-old man presents to the emergency department with a tight bandage around his chest to help reduce pain from a chest wall injury on his right side that occurred during mixed martial arts sparing. Physical exam reveals dullness to percussion, dry crackles and diminished breath sounds over the right lower lobe. Chest x-ray shows elevation of the right hemi-diaphragm. What is the most likely diagnosis? A Atelectasis B Bronchiectasis C Pleural Effusion D Pneumothorax E Pulmonary edema

Atelectasis The patient's injury places him at risk of atelectasis, pneumothorax, or other traumatic injuries. The physical exam and chest x-ray findings are classic for atelectasis (A). Pleural effusion (C) would present with fluid in the costophrenic angle on chest x-ray. Pneumothroax (D) would typically present with findings in the upper lung fields including hyperresonance to percussion. Pulmonary edema (E) would present with increased vascular markings and evidence of fluid within the alveolar space on chest x-ray.

A 22-year-old man is brought to the emergency department by paramedics after having sustained a single stab wound along the left sternal border at the fourth intercostal space. Upon arrival to the emergency department, he was hypotensive and tachycardic. The neck veins were distended and heart sounds were muffled. Which of the following interventions is the most appropriate first-line management of this patient? A Left tube thoracostomy B Pericardiocentesis C Fluid resuscitation D Immediate intubation

B Cardiac tamponade is classically described by the triad of jugular venous distension (JVD), arterial hypotension, and muffled heart sounds. In the emergency department, suspicion of this clinically entity is usually confirmed by ultrasonography and is acutely treated by pericardiocentesis, which will be diagnostic, therapeutic, and buy time until a definitive procedure can be done. A left tube thoracostomy may be indicated in this patient but would not relieve symptoms. Fluid resuscitation though applied to all trauma patients would help stabilize the patient until more therapeutic interventions could be completed. Immediate intubation, even if indicated, would require a prophylactic tube thoracostomy to prevent the development of tension pneumothorax in the event of an unrecognized lung injury. Emergency thoracotomy will relieve the signs and symptoms associated with cardiac tamponade and allow for repair of any underlying cardiac injuries.

Which medication is the treatment of choice for symptomatic patients with hypertrophic cardiomyopathy? A Calcium channel blockers B Nitrates C Thiazide diuretics D Alpha antagonists E Beta-blockers

BB E The use of beta-blockers in symptomatic hypertrophic cardiomyopathy is useful for gaining rate control. This will allow for the optimal amount of filling in order to maintain enough of an ejection fraction.

A 65-year-old recent alcoholic comes to the emergency department with recent onset of dyspnea, with exertion, 3 pillow orthopnea, lower extremity edema, and palpitations, in which he describes his heart as racing. Which of the following is most likely to be the cause of his high-output congestive heart failure? A Mitral regurgitation B Aortic stenosis C Uncontrolled hypertension D Ruptured chordae tendinae E Beriberi

Beriberi Choice E, beriberi, also known as thiamine deficiency, is common among alcoholics, and the only high-output cause of congestive heart failure among the choices offered. Other causes include severe anemia, thyrotoxicosis, and arteriovenouis shunting (for example, in hemodialysis patients). Choice A, mitral regurgitation, is a cause of excessive preload, leading to heart failure. Choice D, ruptured chordate tendinae associated with mitral regurgitation, would also be a cause of excessive preload, leading to heart failure. Choices B and C, aortic stenosis and uncontrolled hypertension, are causes in which too much afterload leads to heart failure

A patient is brought to the Emergency Department by ambulance. He is a 27-year-old male who is well known to the paramedic team as a heroin addict. He is arousable and does not remain alert when aroused. Which of the following physical signs would help to confirm the diagnosis of opioid intoxication? A Bradycardia B Diaphoresis C Mydriasis D Rhinorrhea E Tachypnea

Bradycardia Bradycardia (A), lowered respiratory rate, miosis and somnolence are the main effects of opiates. As with most medications, withdrawal effects are the opposite of the effects of overdose. Tachycardia, tachypnea, rhinorrhea and diaphoresis (B, D, & E) can all occur in opiate withdrawal. Mydriasis (C) is common with other substances of abuse, such as cocaine and LSD.

A 48-year-old man is brought to the emergency department by his sister after suffering from loss of consciousness, followed by muscle rigidity and rhythmic contractions, and then a return to a normal state. When asked about medication use, the patient states he is currently being treated with a drug for depression but cannot remember the name. He claims that he has never had a seizure or seizure-like activity prior to this event. Approximately 6 hours after the first episode, the patient suffers a second one while still in the ED. Which of the following medications is the patient most likely taking? A bupropion B duloxetine C fluoxetine D nortriptyline E phenelzine

Bupropion Bupropion has been shown in some patients to cause seizures in a dose-dependent fashion, particularly in those with a history of head trauma or electrolyte abnormalities. Tricyclic antidepressants (eg, nortriptyline), selective serotonin reuptake inhibitors (eg, fluoxetine), serotonin-norepinephrine reuptake inhibitors (eg, duloxetine), and monoamine oxidase inhibitors (eg, phenelzine) have not been associated with seizures.

A 27-year-old female presents to the emergency department after a motor vehicle accident. Imaging of the left lower extremity shows the following fracture pattern: How would this fracture be described? A No displacement B Complete dorsal displacement C Fifty percent dorsal displacement D Displacement with shortening

C This Fracture is displaced, pertaining to any deviation from anatomical position or alignment to the extent to which the fracture fragments are nonconcentric or offset from each other. This fracture is only 50 percent displaced without angulation or shortening. The magnitude of displacement is expressed in either terms of measurement (i.e. incomplete) or percentage (i.e. 50%) ) of the width of the bone. The direction of displacement is based on the position of the distal fragment relative to the proximal fragment. Separation is the distance two fragments have been pulled apart. Shortening (D) is the amount the bone's length has been reduced, which is not seen in this fracture. Angulation is the degree of "bending" that makes the fragments unparalleled. This would not be described as "no displacement" (A) or "complete displacement" (B).

Out of all cervical vertebrae, which two are responsible for the greatest amount of rotation? A C1 & C2 B C2 & C3 C C3 & C4 D C4 & C5 E C5 & C6

C1 & C2 A Approximately 50% of cervical rotation takes place between the C1 (atlas) and C2 (axis) vertebrae. These first two cervical vertebrae have a different shape from the other cervical vertebrae that allow for this greater range of motion. The remaining 50 % of cervical rotation is split fairly evenly between the remaining vertebrae. Approximately 50 % of flexion and extension occurs between the occiput at the base of the skull and C1 with the remaining 50% distributed fairly evenly between the remaining vertebrae with a slightly higher percentage occurring at the C5 & C6 level.

A 16-year-old boy sustained a head injury during wrestling practice. During a practice match, the boy's head struck the concrete floor and he then received a secondary blow to the head by nearby wrestler. He briefly lost consciousness and then was dazed, confused, and slow to respond to questions for a few minutes. His vision became briefly blurry and double. His balance was initially unstable but improved with time; however, he developed lethargy and a headache. He did not have any nausea or vomiting. His mother was instructed to have him checked for a head injury. On exam about an hour later, he appears tired but is alert and oriented and able to recount the events leading to his injury, though he is a somewhat unsure about what happened right after. His vitals are stable, and visual acuity is 20/40 in both eyes; head and neurologic exam are normal, including equal and reactive pupils and cranial nerves. His very anxious mother says he suffered a concussion 2 seasons ago while playing football and had to sit out practice for 2 weeks. Question What management options would be best for this patient? Answer Choices 1 Observation only 2 Cranial CT scan 3 MRI 4 Skull radiographs 5 Ultrasonography

CT Cranial CT scanning, after a thorough history, physical, and neurologic examination, may be used in addition to observation in the management of children with minor closed head injury with loss of consciousness. Loss of consciousness is uncommon but is associated with an increased risk for intracranial injury. Some studies have suggested that a loss of consciousness, amnesia at the time of evaluation, or headache or vomiting has a prevalence of intracranial injury detectable on CT ranging from 0% to 7%. Furthermore, neurosurgical intervention may be required in 2% to 5% of those. Compared to MRI scans, CT scans are more sensitive for hyperacute and acute intracranial hemorrhage and are more quickly, easily, and cheaply performed. In the absence of available CT scanning, MRI scanning may be an acceptable option. Consensus is that observation alone (after a thorough history and physical plus normal neurologic exam are performed in the clinic, office, ED, or home under care of a competent observer) can be used as the primary management strategy in patients who present with no loss of consciousness, amnesia, vomiting, headache, or mental status changes. Parents or guardians would require careful instructions about when to seek medical attention for a worsening condition. A careful assessment of the caregiver's compliance with instructions must be made and may differ in individual cases; other factors to weigh include distance and time it would take to reach appropriate medical care if the patient's condition worsened. Skull radiographs have a very limited role in head injury evaluation of the child with loss of consciousness. A substantial rate of false-positive results where a fracture is detected without intracranial injury, and the low prevalence of intracranial injury among this specific subset of patients, leads to a low predictive value of skull radiographs. Furthermore, intracranial injury can also occur in the absence of a skull fracture. If imaging is desired, CT scanning is the modality of choice based on increased sensitivity and specificity. The use of ultrasound in traumatic brain injury is being evaluated to measure the optic nerve distensible sheath diameter. This method can predict raised intracranial pressure.

Patients diagnosed with an auricular hematoma are at increased risk of developing which condition? A Cartilage necrosis B Cholesteatoma C Coagulopathy D Exostosis E Otomycosis

Cholesteatoma Auricular hematomas occupy the subperichondral space, leading to decreased or absent diffusion from the perichondrium to the cartilage and resulting in increased risk of necrosis (A). Coagulopathy (C) may predispose a patient to experiencing a hematoma. Cholesteatoma (B) may result from TM trauma, but not blunt trauma to the outer ear.

You are performing a mental status exam on a 19-year-old male patient who was brought to the Emergency Department because of bizarre behavior. As you are speaking with him he keeps repeating words that rhyme with words that either you or he say, but make no sense and are unrelated to your questions. How will you document this behavior in his chart? A Circumstantiality B Clanging C Flight of ideas D Perseveration E Tangentiality

Clanging Word association based on rhyme is called clang association (B) and may be seen in psychotic disorders. Circumstantiality (A) is a disturbance in fluency where the speaker meanders on many side topics before returning to the topic at hand. Flight of ideas (C) is a rapid transition from thought to thought, leading to the speaker losing track of the original idea. Perseveration (D) is a fixed focus returning again and again to the same thought. Tangentiality (E) refers to a disturbance in continuity where the speaker shifts from one thought to another that may be only vaguely related.

A 68-year-old woman with a history of hypertension and diabetes mellitus type 2 comes to the emergency department with her son, who noticed that while decorating for Christmas she seemed more dyspneic than normal, and had to sit down frequently. In addition, he noticed that she was pale and diaphoretic, and insisted on driving her to the emergency department. On questioning, she denies chest pain, but admits to being more fatigued than usual, with frequent jaw discomfort during activity. Activities such as vacuuming her house cause dyspnea, and she now has to stop several times while carrying laundry up from the basement. On physical examination, the patient's blood pressure is 90/50, pulse 99 bpm, respirations 22, and she is afebrile. Auscultation of the chest demonstrates a new systolic murmur. An EKG demonstrates normal sinus rhythm with nonspecific ST and T wave changes. Serial troponin elevations above the 99 th percentile of normal are noted. Which of the following would be the most appropriate next step in the management of this patient? A Clopidogrel, heparin, and aspirin, followed by cardiac catheterization B Nuclear stress test C Treadmill stress test D Thiazide diuretics and loop diuretics

Clopidogrel, heparin, and aspirin, followed by cardiac catheterization In patients with non-ST-segment myocardial infarction, such as this patient with ischemic symptoms and serial troponin elevation above the 99 th percentile of normal, clopidogrel, aspirin, and heparin prior to cardiac catheterization are recommended, with the intention of percutaneous coronary intervention. In addition, morphine, oxygen, nitrates, beta blockade, ACE inhibitors, statin therapy, and glycoprotein IIb/IIa inhibitors should be considered, depending on the patient's blood pressure, heart rate, and hemodynamic stability. Choice D would be useful in the treatment of patients with hypertension and lower extremity edema. Choices B, C, and E, are all forms of stress testing, which should be performed in patients with symptoms of angina pectoris, but not for patients with acute myocardial infarction

The rotator cuff is comprised of which four muscles? A Scalenes, infraspinatus, teres minor, subscapularis B Supraspinatus, infraspinatus, teres major, subscapularis C Supraspinatus, infraspinatus, teres minor, soleus D Supraspinatus, infraspinatus, teres minor, subscapularis E Supraspinatus, intercostals, teres minor, subscapularis

D Supraspinatus, infraspinatus, teres minor, subscapularis D Only option D lists the correct muscles of the rotator cuff located in the shoulder area that contribute to arm elevation, internal rotation and external rotation. Of the teres muscle group, it is the teres minor, not teres major that is part of the rotator cuff. The scalene muscles are in the neck and cause rib elevation and neck movement. The soleus muscles cause plantar flexion of the feet and are located in the calf area of the posterior lower leg.

A teenage girl presents to the emergency department with her parents. She has had symptoms of a urinary tract infection for the last two days, but did not tell her parents until today. She is not sexually active. Today, the girl also has diffuse abdominal pain with vomiting, general malaise, and difficulty breathing. She has no significant past medical history. Her physical exam reveals sinus tachycardia and deep fast respirations with no localization of abdominal pain or rebound tenderness. Initial lab test results reveal a plasma glucose = 378 mg/dL and serum bicarbonate = 14 mEq/L. What is the most likely diagnosis? A Hyperosmolar nonketotic hyperglycemia B Diabetic ketoacidosis C Chronic corticosteroid use D Gestational diabetes E Schmidt syndrome

DKA The correct choice is B, diabetic ketoacidosis (DKA). Classic signs and symptoms of this disorder include polyuria, polydipsia, marked fatigue, nausea, vomiting, signs of dehydration, fruity breath odor, postural hypotension, Kussmaul respirations, and possibly mental stupor or coma. Patients with type 1 diabetes mellitus may present for the first time in DKA. DKA is commonly precipitated by a recent infection. Choice A, hyperosmolar nonketotic hyperglycemia, is seen in patients with type 2 diabetes and presents with extremely high plasma glucose levels without acidosis and ketosis. Choice C, chronic corticosteroid use, can cause hyperglycemia and possible glucose intolerance or diabetes mellitus. This patient has no history of oral corticosteroid use. Choice D, gestational diabetes, occurs in women who are pregnant. Choice E, Schmidt syndrome, is an autoimmune polyglandular syndrome which includes diabetes mellitus in greater than 50% of patients.

A 23-year-old man, unrestrained driver, is brought to the emergency department by ambulance after having been involved in an automobile accident. His vitals are BP: 99/54 mm Hg, P: 112/min, R: 18/min, oxygen saturation: 99%, T: 99.8°F. Examination reveals mild abdominal tenderness with pain radiating to the right shoulder. What is the most appropriate diagnostic test to order initially? A Computed tomography of the abdomen and pelvis B Diagnostic peritoneal lavage C Flat and upright abdominal radiographs D Diagnostic ultrasound

Diagnostic u/s The initial evaluation of blunt abdominal trauma is by the performance of a FAST (focused assessment with sonography for trauma) ultrasound, which is performed by an emergency department physician or surgeon. CT scan remains an adjunct test in hemodynamically stable patients or in patients in whom further assessment of solid intra-abdominal organs is required.

A 22-year-old recent immigrant from Vietnam, who is 28 weeks pregnant with her first child, presents to the emergency department with complaints of worsening dyspnea and lower extremity edema. She is unable to answer definitively whether or not she has a history of rheumatic fever. On physical examination, a possible opening snap, loud S 1 , and a very soft diastolic rumbling murmur is auscultated. When the patient is placed in the left lateral decubitus position, the murmur is accentuated, and heard best at the apex. With inspiration, the murmur does not increase in amplitude. Which of the following is the most likely finding on echocardiogram? A Tricuspid regurgitation B Tricuspid stenosis C Atrial septal defect D Aortic regurgitation/insufficiency E Mitral stenosis

E Choice E, mitral stenosis, is the most likely finding in this patient, who presents with physical exam findings including a possible opening snap, loud S 1 , and a very soft diastolic rumbling murmur which is heard best at the cardiac apex and accentuated by placing the patient in the left lateral decubitus position. Although rheumatic fever was not positively confirmed, the patient likely did have a history, given that the majority of cases of mitral stenosis are secondary to rheumatic heart disease. Patients from Asia, Central America, and South America are exposed more frequently than their counterparts in more developed countries, where antibiotic use is more common. Choices A and B, tricuspid regurgitation and tricuspid stenosis, are also linked with patients with rheumatic heart disease. The murmur of tricuspid regurgitation, however, is a systolic murmur, which increases with inspiration and is heard best at the left lower sternal border. Tricuspid stenosis presents with a diastolic murmur, and with inspiration the murmur increases. It, too, is heard best at the left lower sternal border. Choice C, an atrial septal defect, if large, could present with similar symptoms of exertional dyspnea secondary to a large shunt, but auscultation would reveal a moderately loud systolic ejection murmur that is heard best in the second and third interspaces. This is secondary to increased pulmonary arterial flow. Choice D, aortic regurgitation/insufficiency, is also a diastolic murmur; however, it is usually a diastolic decrescendo murmur that is heard best at the left sternal border.

Which of the following indicates EKG changes consistent with a suspected pulmonary embolus? A Increased QRS amplitude with tall R waves in limb leads and deep S waves in V1 and V2 B Notched P wave in leads I and II with an increased duration C Prolonged PR interval with peaked T waves D Prolonged QRS duration with QS complex in V1, monophasic R wave in leads I and V6 E Sinus tachycardia with a right ventricular strain pattern, prominent S in lead I, Q wave and inverted T in lead III

E Sinus tachycardia with a right ventricular strain pattern, prominent S in lead I, Q wave and inverted T in lead III Approximately 70% of patients with a pulmonary embolus will demonstrate EKG changes. The changes present may range from sinus tachycardia to findings consistent with marked right heart strain and dilatation. The correct answer also includes the classic "S1Q3T3" pattern of changes associated with pulmonary embolism. The findings in choice A are consistent with left ventricular hypertrophy. Choice B is indicative of left atrial enlargement. Choice C is consistent with hyperkalemia. Choice D indicates left bundle branch block. These changes are not commonly associated with pulmonary embolism.

A 21-year-old man presents to the emergency department complaining of a swollen left ear (see image) after he experienced blunt trauma in a collegiate wrestling match. Which of the following is the most appropriate management for this condition? Picture of an auricular hematoma. A Evacuation B Evacuation followed by antibiotics C Evacuation with subsequent antibiotics and splinting D Ice and compression E Reevaluation in 24 hours

Evacuation with subsequent antibiotics and splinting The patient has an auricular hematoma that must be evacuated to prevent cartilage necrosis or infection. Evacuation, antibiotics, and splinting (C) remove the hematoma, minimize the risk of infection, and help prevent the re-accumulation of blood.

A disk herniation that is putting pressure on the L5 nerve root may present with weakness of what muscle(s)? A Anterior tibialis B Extensor hallucis longus C Gastrocnemius-soleus D Iliopsoas E Peroneus longus and brevis

Extensor hallucis longus The extensor hallucis longus muscle's motor function is associated the L5 motor neuron, which also supplies the gluteus medius and extensor digitorum longus and brevis muscles. The anterior tibialis muscle is supplied by the L4 motor neuron. Nerves emanating from T12, L1, L2 and L3 supply the iliopsoas. Gastrocnemius, soleus and peroneus longus and brevis are all supplied by nerves coming from the S1 area. The plantar flexing gastrocnemius and soleus muscles also are supplied by S2.

A 3 year-old boy is brought to the emergency department due to acute onset of cough and wheezing. Physical exam reveals focal wheezing in the right lower lobe. Which of the following is the most likely diagnosis? A Asthma B Bronchiectasis C Epiglottitis D Foreign body aspiration E Vocal cord dysfunction

FB aspiration The acute onset of symptoms and their localization to the right lower lobe are consistent with foreign body aspiration (D). Asthma (A) presents with diffuse wheezing. Bronchiectasis is less common in this age group and typically presents with a copious amount of sputum production. Epiglottis (C) and vocal cord dysfunction (E) present with upper airway symptoms.

A 2-year-old child presents to the emergency department via ambulance due to a seizure lasting approximately 2 minutes with jerking and somnolence. En route in the ambulance her vital signs are: temperature 39°C rectal; pulse 120/min; respirations 32/min; blood pressure 110/64 mm Hg. Upon further questioning, her mother claimed she had a runny nose yesterday. On physical examination, she is sleepy but arousable with negative Kernig and Brudzinski signs. Which of the following seizures is the MOST likely diagnosis? A absence seizure B complex partial seizure C febrile seizure D simple partial seizure

Febrile Seizure C A febrile seizure is a brief (less than 15 minutes), generalized, symmetric, tonic-clonic seizure associated with a febrile illness (temperature greater than 38.8°C) without any central nervous system infection or neurologic cause. An absence (petit mal) seizure is a brief (2 to 25 seconds) loss of consciousness that can occur multiple times per day. There is no loss of tone, and frequently the only observable behaviors are staring or minor movements such as lip smacking and semipurposeful movements of the hands. There is no postictal period. Complex partial seizures (psychomotor) have varied symptoms including alterations in consciousness, unresponsiveness, and repetitive complex motor activities that are purposeless. Often, at the beginning of the attack, there is a psychoillusory phenomenon such as hallucinations, visual distortions, visceral sensations, or feelings of intense emotions. Simple partial seizures include focal motor, adversive, and somatosensory seizures. Manifestations of these seizures are varied including hallucinatory, psychoillusory, or complex emotional phenomena. Children will interact normally with their environment, with the exception of those limitations imposed by the seizure. Following the seizure (minutes to hours), there may be transient paralysis of the affected body part.

An 8-month-old female is diagnosed with respiratory syncytial virus bronchiolitis while in the emergency department. Which of the following strongly indicates a need for admission to the hospital and continued monitoring? A Age of 8 months B Birth at 38 weeks C Feeding difficulty with decreased oxygen saturation D Oxygen saturation of 96%

Feeding difficulty with decreased O2 sat C Brochiolitis patients must be considered at risk of developing severe disease and/or apnea when certain criteria are present, thus requiring admission. This includes, but may not be limited to, the following: birth <37 weeks gestation, age <12 weeks, witnessed apnea, underlying cardiopulmonary disease, immunodeficiency, tachypnea based on expected respiratory rate per age, decreased oral intake or feeding difficulty with associated decreased oxygen saturation, decreased oxygen saturation with varying ranges based on source (most being <95%), a history of previous intubation, and a caregiver ability to adequately provide care and monitoring.

A 40- year-old female patient comes to the office because she notices that she is easily fatigued but cannot pinpoint a direct cause of her fatigue. She has experienced muscle tenderness to the shoulders and other large muscle groups, sensitivity to touch these areas, and has also felt depressed lately. Based on this history and the vague physical exam findings, what is the most likely diagnosis? A Fibromyalgia B Polyarteritis nodosa C Polymyositis D Scleroderma

Fibromyalgia A Fibromyalgia syndrome (FMS) primarily affects woman between the ages of 20 and 60 and is now the second most common condition seen in rheumatologists' offices behind only rheumatoid arthritis. This condition spares the joints, but causes tender areas throughout the soft tissue as well as generalized pain and fatigue. There are several common trigger points, both anteriorly and posteriorly that help confirm the diagnosis. Polyarteritis nodosa is a systemic necrotizing vasculitis that generally affects medium size muscular arteries. Patients can present with many of the same symptoms as those with fibromyalgia (fatigue, muscle pain, and others), but these patients often have skin manifestations, joint pain, and fever, along with renal, GI, and cardiovascular signs not seen in fibromyalgia. Polymyositis is an inflammatory condition of the muscles that is characterized by proximal muscle weakness, with much less pain than is seen in fibromyalgia. Despite the proximal muscle weakness there is generally not significant atrophy. Sjogren's syndrome does not generally present with any muscular manifestations. It is a chronic inflammatory disorder that affects the salivary and lacrimal glands resulting in dry eyes and a dry mouth. Scleroderma is classically associated with thickened and hardened skin, but it can have internal organ involvement as well affecting many different body systems. When it affects the musculoskeletal system it tends to affect joints and the areas where tendons cross joints. It can cause contractures, pain, and swelling as well as fatigue and weakness

A 46-year-old female presents with pain to her left wrist. She complains that it is painful and swollen as she points to the volar aspect of the wrist on the radial side. On examination, there is a small, soft bump on the dorsum of her wrist with a jelly-like consistency. What is the most likely diagnosis? A Cancerous tumor B Fracture C Ganglion cyst D Hematoma E Lipoma

Ganglion Cyst C Ganglion cysts commonly occur on the dorsal or volar aspect of the wrist. They result when a joint capsule or tendon sheath is damaged, allowing synovial fluid to escape producing a one-way valve, which allows fluid into the cyst, but not back out. The accumulating fluid forms the ganglion cyst. These cysts may or may not be tender and can fluctuate in size depending on activity level of the affected extremity. Cancerous tumors would tend to be much more firm, but also may be relatively pain free. Fractures would generally be exquisitely tender and if the bump is due to a displaced bone, it would be much more firm than a ganglion cyst. Hematomas are generally associated with acute trauma and would be tender and ecchymotic in many situations. Lipomas are benign fatty tumors that are more commonly seen on the thenar eminence than the dorsum of the wrist and their size does not change based on activity level.

A 9-year-old female presents to the emergency department after falling out of a second story window. Imaging of the left upper extremity shows the following fracture pattern: How would this fracture be described? A Transverse B Oblique C Spiral D Segmental E Greenstick

Greenstick A greenstick fracture occurs almost exclusively in young children as a result of their bones being more pliable than those of adults. On imaging, a greenstick fracture has cortical disruption on the convex side of the bone with an intact periosteum on the concave side of the fracture. Spiral fractures (C) are often seen in non-accidental or child abuse cases. Transverse (A), segmental (D), and oblique (B) fractures involve both cortices.

A 23-year-old man presents to the outpatient clinic for follow-up from a recent urgent care visit. He complains of sore throat, fever, fatigue, myalgias, and a rash that started 5 days ago, and have worsened since he was seen in the urgent care 3 days ago. The patient appears non-toxic with a temperature of 39.4 degrees Celsius. Physical exam reveals pharyngeal and tonsillar erythema without exudates, generalized lymphadenopathy, a morbilliform rash on his trunk, and no hepatosplenomegaly. A rapid strep screen and Monospot performed at the local urgent care were reportedly negative. Which of the following laboratory tests is most likely to confirm the expected diagnosis? A Complete blood count B Cytomegalovirus titer C Group A beta-hemolytic strep culture D HIV viral load E Epstein-Barr virus titer

HIV viral load The patient presentation is consistent with acute retroviral syndrome, which is best confirmed during this initial presentation phase through direct testing for the HIV virus, such as an HIV viral load (D). The lack of tonsillar exudates, a negative monospot, and presence of generalized adenopathy make infectious mononucleosis (B and E) less likely. A CBC (A) may show lymphopenia and support the diagnosis, but it doesn't confirm the diagnosis.

A 27-year-old African American female presents to the emergency department with low blood pressure of 100/40, palpitations, and shortness of breath. She is currently under treatment for Wolff-Parkinson-White syndrome and has been taking procainamide for the last two years. An electrocardiogram is obtained on the monitor and reveals the rhythm strip shown. What is the treatment of choice for this patient? EKG shows tosades de pointe A Intravenous calcium B Intravenous magnesium C Oral potassium D Subcutaneous epinephrine E Metoprolol

IV Mg B The rhythm strip reveals ventricular tachycardia in the form of torsades de pointes. In this case, the primary medical intervention is to administer magnesium sulfate to counter the irregular activity. Antiarrhythmics, antidepressants, and some antibiotics can be responsible for this arrhythmia. In addition to the magnesium, administration of beta-blockers can also be helpful.

A 55-year-old woman with a history of mitral valve replacement and mitral stenosis (secondary to rheumatic heart disease) presents to the emergency department with increasing dyspnea while walking up one flight of stairs. She denies chest pain and discomfort, but states that for the past few weeks she has noticed palpitations. She also admits to lower extremity edema, which is new within the last week. On EKG, she demonstrates atrial flutter with 2:1 AV block. Her INRs have been therapeutic for the past 4 weeks. Which of the following is the most appropriate next step in treating this patient? A IV quinidine B IV ibutilide C IV vasotec D IV amiodarone E IV dopamine

IV ibutilide Choice B, IV ibutilide, has been found to be most effective in converting atrial flutter to sinus rhythm out of all the choices listed. Choice A is contraindicated, as the atrial conduction may decrease to the point that 1:1 atrial to ventricular conduction can occur with the administration of class I antiarrhythmics. The ventricular rate can then increase to rates greater than 200 bpm, and hemodynamic collapse may occur. Choice C is useful for blood pressure control, but not for heart rate control. Choice D is useful for chronic atrial flutter heart rate management, or for helping to maintain sinus rhythm after cardioversion has occurred. Choice E is useful for pressor support, and not for heart rate control or conversion to normal sinus rhythm.

A 55-year-old woman with a history of hypertension and 2 vessel CABG presents to the emergency department with increasing dyspnea while walking up one flight of stairs. She denies chest pain and discomfort, but states that for the last 24 hours she has also noticed palpitations. On physical examination, her vital signs are stable. On EKG, she demonstrates atrial flutter with 2:1 AV block. Her echocardiogram demonstrates normal LV systolic function and normal valvular function. Which of the following is the most appropriate therapy for this patient? A IV ibutilide after 4 weeks of anticoagulation with warfarin B IV ibutilide alone C IV quinidine after 4 weeks of anticoagulation with warfarin D IV quinidine alone E IV dopamine

IV ibutilide alone Choice B, IV ibutilide, is the most appropriate choice for this patient. Therapy for patients with atrial flutter and atrial fibrillation is the same in regards to anticoagulation; therefore, in a patient with a CHADS2 score of 1 and with symptoms of less than 48 hours duration, cardioversion to normal sinus rhythm, whether chemically or electrically, is recommended. Out of all the choices listed, IV ibutilide has been found to be most effective in converting atrial flutter to sinus rhythm. Choices C and D are contraindicated, regardless of the type of anticoagulation paired with it, as quinidine is a class I antiarrhythmic. The atrial conduction may decrease to the point that 1:1 atrial to ventricular conduction can occur with the administration of class I antiarrhythmics. The ventricular rate can then increase to rates greater than 200 bpm, and hemodynamic collapse may occur. Choice E is useful for pressor support, which is not indicated in this patient who is quite stable.

A 65-year-old recent alcoholic comes to the emergency department with recent onset of dyspnea with exertion, 3 pillow orthopnea, lower extremity edema, and palpitations, in which he describes his heart as racing. Which of the following is the most appropriate treatment for his high-output congestive heart failure? A IV dextrose alone B IV thiamine C IV enalapril D IV dopamine E IV diltiazem

IV thiamine Choice B is the most appropriate treatment, as the patient is demonstrating high output congestive heart failure secondary to beriberi, or thiamine deficiency. In 50% of patients, IV thiamine administration, along with other vitamins and glucose, will resolve the patient's symptoms. Choice A, IV dextrose alone in patients with very low thiamine stores, can worsen signs and symptoms of heart failure. Choice C, IV enalapril, is appropriate therapy for patients in need of better blood pressure control, and as an ACE inhibitor, in patients with left ventricular systolic dysfunction, which is not the cause of this patient's heart failure. Choice D, IV dopamine, is useful in patients in need of pressor support, but will not help treat high-output heart failure secondary to thiamine deficiency. Choice E, IV diltiazem, is useful for heart rate control in patients with atrial fibrillation with a rapid ventricular rate.

A 3 year-old boy is brought to the emergency department due to acute onset of cough and wheezing. Physical exam reveals focal wheezing in the right lower lobe. Which of the following should be ordered to confirm the suspected diagnosis? A Arterial blood gas B Inspiratory and forced expiratory chest x-rays C PA and lateral chest x-ray D Peak expiratory flow rate E Spirometry

Inspiratory and forced expiratory chest x-rays The patient most likely has aspirated a foreign body. This is best evaluated through the demonstration of inspiratory localized hyperinflation and expiratory mediastinal shift (B) on chest x-ray. ABG (A) results will vary depending on the severity of airway obstruction. PA and lateral chest x-rays (C) are typically normal. PEFR (D) and Spirometry (E) are not typically able to accurately assess this localized airway obstruction.

A 43-year-old male presents to the Emergency Department complaining of right eye pain after treating his yard with fertilizer and lime. He attempted to flush his eye at home without relief of pain. Which of the following is the most appropriate initial step in managing this patient's symptoms? A Double evert his eyelids to look for remaining foreign bodies B Fluorescein stain his eye C Instill proparacaine 0.5% ophthalmic solution D Irrigate his eye until the pH is between 6.8 and 7.4 E Refer to ophthalomogist

Instill proparacaine 0.5% ophthalmic solution The patient requires all of the above steps and should be given pain relief (C) prior to thoroughly flushing the eye (D), removing foreign bodies (A), assessing for corneal injuries (B), and referring to ophthalmology (E).

A soft tissue neck x-ray of a patient who complains of a progressively worsening sore throat reveals this lateral film (see image). Based on these findings, what is the initial treatment of choice for this patient? A Endotracheal intubation B Intravenous steroids C Ribovirin injection D Incision and drainage E Cricoidotomy

Intravenous steroids This case of acute epiglottitis is treated with immediate intravenous steroids. Provided that the patient is able to maintain the airway and also keep oxygen saturation rates above 92%, the patient can improve with steroids and supportive care. Antiviral medications have little effect on the overall illness.

A 14-year-old boy presents to the emergency department with his parents. He has a history of type 1 diabetes, and has had bronchitis for the last few days. He is now presenting with difficulty breathing, worsening fatigue, polydipsia, and polyuria. His last fingerstick glucose at home this morning was 350 mg/dL. Which of the following patterns of breathing are characteristic of this complication of diabetes? A Cheyne stokes respiration B Bradypnea C Biot breathing D Kussmaul breathing

Kussmaul breathing The correct choice is D, Kussmaul breathing, which is deep regular breathing or hyperpnea. It can be seen as a compensatory action of metabolic acidosis and hypoxia. Choice A, Cheyne-Stokes respiration, is a waxing and waning pattern of rate and volume that includes periods of apnea. This can be seen in patients at high altitudes, and with severe left sided heart failure or neurologic disease. Choice B, bradypnea, is noted with a slower than usual respiratory rate and can be seen with use of CNS depressant drugs, uremia, or structural intracranial lesion. Choice C, Biot breathing, is an uncommon variant of Cheyne-Stokes respiration, with periods of apnea alternating with a series of equal breaths that end abruptly. It can be seen in patients with meningitis. Choice E, painful respiration, is relatively normal in pattern, but interrupted by pain during breathing from such disorders as pleurisy, fractured ribs, or subphrenic inflammation.

Which of the following is the drug of choice for acute hypertensive encephalopathy? A Labetolol B Clonidine C Furosemide D Nifedipine E Nitroglycerin

Labetolol Neurologic emergencies associated with elevated blood pressure must be throurougly evaluated to determine the diagnosis and appropriate treatment plan. In the case of hypertensive encephalopathy, immediate attention must be focused on blood pressure reduction. Multiple agents are known to decrease blood pressure, but selection must also focus on how rapidly each agent works, how titratable the agent is, and any potential sequelae from using a particular agent. Sodium nitroprusside was classically the agent of choice for rapid blood pressure management, but it has fallen out of favor due to its monitoring requirements and rate of toxicity. Labetolol, a beta-blocker, is recommended for acute management, except in the case of cocaine intoxication. It has a rapid onset of action, can be titrated, and is given IV. Use of labetolol should be avoided in patients with known asthma, COPD, congestive heart failure, bradycardia, and second or third degree heart block. Additional agents appropriate for hypertensive encephalopathy include enalaprilat, esmolol, fenoldopam, hydralazine, and nicardipine, with each being considered carefully (based on patient condition and other factors). Agents with known CNS adverse effects, such as clonidine, should be avoided. Use of clonidine must also be monitored due to potential rebound hypertension. Nifedipine use is discouraged in hypertensive emergencies. Nitroglycerin should not be used for hypertensive encephalopathy because it increases intracranial pressure (additional information is available in Table 61-4).

Treatment for alcohol withdrawal

Librium and Ativan

A 7-year-old is diagnosed with an acute case of hematogenous osteomyelitis accompanied with fever and leukocytosis. Based on your knowledge of the disease, which bone is most likely to present with the infection? A Feet B Hands C Long bones D Pelvis E Vertebrae

Long bones Fortunately, hematogenous osteomyelitis is not common in children, but when it does occur it primarily is found in the long bones. The femur, tibia and humerus are the most typical locations for osteomyelitis in children. The highly vascular metaphysis of long bones contribute to the potential for hematogenous spread of the implicated pathogen. Osteomyelitis can occur at any of the locations mentioned in the answer choices given, but at a significantly lower rate than in the long bones. The rate of occurrence at several selected locations is given below: Feet - 9% Femur 25% Hands - 6% Humerus - 13% Pelvis - 8% Radius/ulna - 6% Tibia/fibula - 28% Vertebrae - 2%

A 24-year-old male is brought to the emergency department by his girlfriend. She states that he began having a seizure in the car approximately seven minutes ago. She is not sure of his medications, but states he has a known seizure history and has seizures a few times a year. The patient is currently actively seizing. Which of the following is the first-line agent to give this patient? A Carbamazepine B Gabapentin C Levetiracetam D Lorazepam E Phenytoin

Lorazepam D This patient has a known history of seizures, with current ongoing seizure activity and the concern of status epilepticus, a true emergency. Prolonged seizure activity is associated with hyperthermia, metabolic disturbances, cardiopulmonary dysfunction, and irreversible damage. Lorazepam, a benzodiazepine, increases the activity of gamma-aminobutyric acid (GABA) in the brain, thereby calming the excessive electrical nerve activity related to the seizure and slowing the central nervous system. It is the preferred first line agent for temporary control of seizure activity due to the duration of action of approximately 12 to 24 hours. This allows additional therapeutic agents to be administered while gaining control of the seizure activity. The other agents are antiepileptic medications, which are used for long-term seizure control. They are differentiated by efficacy, convenience, side effects, and drug-drug interactions.

Which physical complaint is the leading cause of lost work time and disability for patients under the age of 45? A Ankle pain B Hip pain C Knee pain D Low back pain E Shoulder pain

Low back pain D Low back pain is the most common causes of lost time from work and disability. Factors that can lead to back problems include repeated or heavy lifting and twisting or use of vibrating equipment along with poor fitness levels, smoking, poor job satisfaction and other psychosocial factors. Fortunately, 80% of patients experience significant recovery within one month. All of the other types of pain can occur based on multiple factors, but none of them occur with a frequency that rivals low back pain

A 62-year-old man is brought to the emergency department after being found unresponsive in his car. On physical examination, his pupils are noted to be 7 mm on the right and 3 mm on the left. Which of the following diagnostic tests is most likely to identify the cause of the patient's signs and symptoms? A CBC with differential B serum electrolytes C MRI with contrast D liver function tests E skull X-rays

MRI with contrast C The patient's unilateral symptoms are best explained by a local anatomical cause (e.g., tumor) that would be detected with an imaging study (MRI). An MRI is preferred over skull X-rays to assess directly for intracranial pathology. CNS abnormalities arising from systemic causes are more likely to be symmetric.

What types of connective tissue are injured in a strain? A Bones and muscles B Fascia and joint capsules C Ligaments and joint capsules D Muscles and tendons E Tendons and bones

Muscles and tendons A strain involves injury to the muscles and tendons that are responsible for active movement of various body parts. Fascia is a part of the muscle-tendon unit, so injury to fascia would be considered a strain as well. Injury to ligaments and joint capsules would be considered a sprain and damage to bone would be classified as a fracture.

An 18-year-old woman is transferred to your emergency department from a local college infirmary. She presented yesterday with a complaint of headache but became confused and is now febrile. You notice a petechial rash on physical examination and her cerebrospinal fluid comes back with increased WBCs, increased protein, and decreased glucose. What is the most likely organism responsible for her meningitis? A Haemophilus influenzae B cytomegalovirus C Neisseria meningitidis D Mycobacterium tuberculosis E coxsackievirus B

Neisseria meningitis C Neisseria meningitidis and Streptococcus pneumoniae are the most common etiologic agents for bacterial meningitis in this patient's age group. So much so that many colleges and universities require a vaccine for students who live in dormitories. Her fever and the cerebrospinal fluid values are consistent with a bacterial and not a viral infectious source for the meningeal irritation.

A 19 year-old-male is brought to the emergency department due to psychosis. During the evaluation, he is noted to appear unkempt and has reduced motor activity. His affect is flattened, and he is using made-up words and phrases during speech. This patient is exhibiting what type of verbal utterance? A Echolalia B Hallucination C Neologism D Catatonia E Verbigeration

Neologism C The use of made-up words or phrases is termed neologism. Echolalia is a repetition of words spoken by others. Hallucinations are perceptual distortions, and may be auditory or visual. Catatonia describes a state in which a person is withdrawn and shows minimal bodily movement, or one in which there is severe excitement and the person shows purposeless and stereotyped movements. Verbigeration is the repetition of senseless phrases or words.

A 56-year-old male, with history of hyperlipidemia and non-insulin-dependent diabetes mellitus (NIDDM) presents to the emergency department with a history of increasing peripheral edema over the past week. On examination he is noted to have periorbital, scrotal, and +2 pretibial edema. His lungs are CTAB. He denies any chest pain or shortness of breath. Urine dipstick reveals 4+ protein. Urine microscopic reveals Maltese crosses consistent with lipiduria. Labs include a decreased serum albumin of 2 g/dl, decreased total protein of 5.5 g/dl, and normal glomerular filtration rate (GFR). What is the most likely diagnosis? A pyelonephritis B congestive heart failure (CHF) C nephrotic syndrome D prostatitis

Nephrotic syndrome C The correct answer is (C). This patient has typical symptoms of nephrotic syndrome, which includes significant proteinuria, hypoalbuminemia, and typical presentation of edema. He also has a history of hyperlipidemia and laboratory findings of lipiduria, which is also common in nephrotic syndrome. Furthermore, his history of diabetes mellitus is also a potential cause of nephrotic syndrome. Pyelonephritis and prostatitis would present with urine WBCs and is not consistent with the laboratory findings or edema. CHF would more likely present with dyspnea, rales on exam, and peripheral edema but would unlikely involve the periorbital area. DVT would likely present with unilateral swelling of the LE, and discomfort and is not consistent with the laboratory findings above.

A 56-year-old right hand dominant male presents with swelling to the right elbow. He explains to you that he has had this type of swelling before and was diagnosed with bursitis. Based on this description, where would the most likely site for the elbow bursitis be located? A Capitellum B Lateral epicondyle C Medial epicondyle D Olecranon E Radial head

Olecranon D The olecranon bursa is very vulnerable to injury and chronic inflammation due to its superficial location on the extensor side of the elbow. Acute falls, chronic pressure, and other inflammatory processes such as rheumatoid arthritis or gout may contribute to olecranon bursitis. The medial and lateral epicondyles are known to be associated with chronic tendonopathies. The radial head is a common location for subluxation or fracture, while the capitellum at the distal end of the humerus is sometimes associated with fractures and articular cartilage damage.

A 66-year-old female with a history of nephrotic syndrome presents to the emergency department complaining of a non-productive cough and dyspnea on exertion following a recent vacation to Orlando. She is currently taking prednisone. What diagnosis is highly suspected in this patient due to her history of nephrotic syndrome? A pneumonia B pulmonary embolism C chronic obstructive pulmonary disease (COPD) exacerbation D Cushing's syndrome E asthma exacerbation

PE The correct answer is (B). Patient's with nephrotic syndrome commonly have a hypercoagulable state and are at risk for deep venous thrombosis (DVT) with resultant PE. Patients on chronic prednisone may have a risk of developing Cushing's syndrome, but this is not consistent with the patient's history. Pneumonia is possible due to chronic prednisone use but not directly caused by her history of nephrotic syndrome. COPD and asthma are not linked to nephrotic syndrome.

A 78-year-old woman with known diabetes mellitus type 2 is brought to the emergency room after a neighbor became concerned when newspapers began piling up on her doorstep and called the police. In the emergency department, she is found to be lethargic and disoriented, with tenting of the skin, sunken eyes, and dry mucous membranes. She is hypotensive and has a rapid pulse. She is wearing a diaper that apparently has been in place for several days, but is barely moist. What is the most appropriate fluid therapy for them to initiate? A 0.45% saline B 0.9% saline C 5% dextrose in water D 5% dextrose in 0.45% saline E Lactated Ringers

0.9% saline This woman is severely dehydrated so 0.9% saline is indicated. If she were less dehydrated, 0.45% saline (A) would be appropriate because of likely hyperosmolality. Once her glucose has dropped to 250 mg/dL, she should receive dextrose in water (C) or 0.45% saline (D) to prevent her glucose level from dropping too low. Lactated Ringers ı is contraindicated in patients who are likely to have severe acidosis or alkalosis.

A 23-year-old patient who has recently been on a ski trip presents with pain to the right hand after sustaining a fall. It is difficult to move, and there is pain on flexion of the digit. Based on this history what ligament would the patient most likely have injured? A 1st MCP joint ulnar collateral ligament B 2nd MCP joint ulnar collateral ligament C 3rd MCP joint ulnar collateral ligament D 4th MCP joint ulnar collateral ligament E 5th MCP joint ulnar collateral ligament

1st MCP joint ulnar collateral ligament The ulnar collateral ligament at the base of the thumb, or 1st MCP joint, is often injured in forced abduction, such as a fall while skiing or during other sporting activities. An injury to this ligament has traditionally been called Gamekeeper's Thumb, but the origin of this term referred to a more chronic injury sustained by English gamekeepers as a result of the way they killed rabbits using their hands. Any of the MCP joint ulnar collateral ligaments could be injured in a fall if the mechanism of injury creates significant forces on the ligaments, but the 1st MCP joint is far more commonly injured than the others mentioned above.

You are getting ready to evaluate a patient who has a past history of gout. The chief complaint is that the patients gout has flared up again, causing pain. Based on your knowledge of gout, which joint is most commonly the site of an initial gout attack? A 1st metatarsaphalangeal joint B Knee C Wrist D Elbow E Ankle

1st MTP joint A Eighty percent of gout attacks affect only one joint. The most common joint to be involved is the 1st metatarsal phalangeal joint. This common phenomenon is called podagra. Gout can affect other joints as well, including the knee, ankle and tarsal joints of the foot. Upper extremities are not commonly affected by gout. The knee is the most common joint to be affected by pseudogout followed by the wrist, MCP joints of the hands, hips, shoulders, elbows and spine

Patients in which of the following age groups are least likely to experience a dislocation or sprain when a significant stress is placed on their joints? A 5-10 years old B 15-20 years old C 35-40 years old D 55-60 years old E 75-80 years old

5 -10 yo Dislocations and ligamentous injuries are uncommon in prepubertal children as the ligaments and joints are quite strong as compared to the adjoining growth plates. Excessive force applied to a child's joint is more likely to cause a fracture through the growth plate than a dislocation or sprain.

A 21 year-old female presents to the emergency department after having a cast applied to her right arm earlier that day. Approximately one hour ago she began having extreme 10/10 pain in her right arm and is in visible distress. When considering the diagnosis of compartment syndrome, permanent damage to the muscle begins after how many hours of ischemia? A >2 hours B >6 hours C >8 hours D >12 hours E >24 hours

> 8 hrs Permanent damage results after >8 hours (C) of ischemia. Nerves begin to lose conduction within 2 hours of onset of elevated pressures. Neurapraxia can occur within 4 hours, and irreversible damage occurs 8 hours after elevated pressures. Functional impairment is unlikely when compartment syndrome is diagnosed and treated within 6 hours of its onset. While deficits can occur prior to 8 hours, permanent damage is usually not seen before 8 hours (A and B). After 12 or 24 hours, permanent damage has already resulted in most cases (D and E). The key to early detection of compartment syndrome is a high index of clinical suspicion.

A 50-year-old woman with a history of hypertension complains of chest tightness and dyspnea while walking up one flight of stairs. She recently experienced an episode of near-syncope. She denies a history or rheumatic fever. On auscultation, a crescendo-decrescendo systolic ejection murmur is heard at the upper right sternal border radiating to the carotids bilaterally. Given the patient's physical exam findings, which of the following is the most likely diagnosis? A Severe aortic stenosis secondary to congenital bicuspid aortic valve B Aortic regurgitation/insufficiency C Mitral stenosis D Aortic regurgitation/insufficiency E Tricuspid regurgitation/insufficiency

A Choice A is correct, as the murmur of aortic stenosis is usually described as a crescendo-decrescendo or systolic ejection murmur heard best at the right upper sternal border. In addition, the murmur of aortic stenosis is frequently transmitted to the carotid arteries. In a patient of this age, with symptoms suggestive of severe aortic stenosis and with these physical exam findings, a congenital bicuspid aortic valve is the best choice, especially as the patient denies a history or rheumatic fever, which is also a cause of developing aortic valve stenosis in individuals under the age of 65. Patients with a congenital bicuspid aortic valve typically develop symptoms once the valve leaflets have become calcified and thickened, secondary to the undue stress over many years on a structurally abnormal aortic valve. Choice B is incorrect, as the murmur of aortic regurgitation is usually described as a high-frequency decrescendo early diastolic murmur heard best at the left upper sternal border or at the right upper sternal border. Choice C is incorrect, as the murmur of mitral stenosis is described as a low-frequency rumbling diastolic murmur that is decrescendo in early diastole, but may become crescendo up to the first heart sound with moderately severe mitral stenosis and sinus rhythm. Choice E is incorrect, as the murmur of tricuspid regurgitation is described as a holosystolic descrescendo murmur.

A 22-year-old male hurts his right knee while playing football on artificial turf. He states that he planted his foot and went to turn, but his leg didn't turn with his body. He instantly felt a popping sensation in the knee. A few hours later he develops an effusion. Based on the history of the injury, which knee structure was likely injured? A Anterior cruciate ligament (ACL) B Lateral collateral ligament (LCL) C Medial collateral ligament (MCL) D Posterior cruciate ligament (PCL) E Quadriceps tendon

ACL A Non-impact rotational or hyperextension forces are the most common mechanisms for sustaining a tear of the ACL. One third of patients report hearing an audible popping sound as their ACL tear occurred. Because the ACL is a vascular structure, when it tears a rapid bloody effusion (hemarthrosis) usually develops which effects mobility of the joint. Lateral collateral ligaments are the least likely to be injured as the type of force necessary to cause injury would be a varus stress which is unlikely to occur in typical circumstances. Medial collateral ligament injuries are fairly common and produced by a valgus force that stresses the ligament. This can occur in many sporting events including those in which another competitor might fall on or dive into the lateral aspect of the knee. Trauma to a knee can result in tears of both the ACL and MCL in certain situations. A tear of the quadriceps tendon usually occurs when a person falls on a knee that is partially flexed. As the quadriceps muscle contract to prevent excessive flexion, the force and momentum of the fall may overwhelm the knee extension mechanism and cause the rupture. No such mechanism occurred in our scenario. Posterior cruciate injuries occur when the tibia is driven posterior in relation to the femur as may happen when a car dashboard is driven into the tibias during a major front impact collision. A powerful hyperextension force can result in both ACL and PCL tears (usually in that order). PCL tears are much more uncommon than ACL tears and don't generally occur with basic rotational forces as described in our patient scenario.

What is the most likely reason for a patient to have cardiogenic shock? A Acute myocardial infarction B Sepsis C Trauma D Aortic dissection E Pericardial effusion

AMI While trauma can account for a patient being subjected to cardiogenic shock, the overwhelming etiology for the shock comes from an MI.

A 23-year-old man presents to the outpatient clinic for follow-up from a recent urgent care visit. He complains of sore throat, fever, fatigue, myalgias, and a rash that started 5 days ago, and have worsened since he was seen in the urgent care 3 days ago. The patient appears non-toxic with a temperature of 39.4 degrees Celsius. Physical exam reveals pharyngeal and tonsillar erythema without exudates, generalized lymphadenopathy, a morbilliform rash on his trunk, and no hepatosplenomegaly. A rapid strep screen and Monospot performed at the local urgent care were reportedly negative. Which of the following prevention strategies should be recommended to this patient? A Abstain from sexual activity B Avoid aspirin C Avoid contact sports and rest D Bedrest and increased fluids E Take the full course of antibiotics

Abstain from sexual activity The patient presentation is consistent with acute retroviral syndrome. The patient is highly contagious and should be counseled on strategies to prevent transmission of HIV to others (A). Aspirin use in viral syndromes (B) is associated with Reye's syndrome, but most often occurs in children with influenza or varicella. Avoiding contact sports (C) is appropriate patient education for a patient with infectious mononucleosis, and patients with group A strep pharyngitis should be instructed to take the full course of their antibiotics (E).

A 66-year-old woman presents to the emergency department with a complaint of abdominal pain and distension for the past 3 days. Examination reveals a protuberant abdomen with diminished bowel sounds and tympany to percussion. Flat and upright abdominal radiographs reveal distended loops of bowel with prominent haustral markings. Which of the following etiologies is the most likely cause of the patient's condition? A Volvulus B Adenocarcinoma C Diverticular disease D Strangulated hernia E Adhesions

Adenocarcinoma Large bowel obstructions are most commonly caused by an adenocarcinoma (65%). This is followed in decreasing incidence by diverticular scarring and volvulus. Adhesions are the most common cause of small-bowel obstruction but are rare as a cause of large bowel obstruction. The presence of haustral markings on radiographic evaluation helps differentiate between small and large bowel involvement.

A 23-year-old female who has a history of supraventricular tachycardia is having an acute episode again. She has attempted a valsalva maneuver without success in breaking the arrhythmia. The ECG confirms SVT. What is the next step in therapy for this patient? A Atropine B Amlodipine C Adenosine D Amiodarone E Metoprolol

Adenosine C Intravenous adenosine is the treatment of choice in this clinical situation. If successful the adenosine will break the cycle of tachycardia, usually with a pause. The initial dose is 6mg, followed by two, 12 mg doses if unsuccessful. While this is the standard protocol as guided by ACLS, additional doses of adenosine can be given in a hospital setting, as the half-life of the drug is extremely short. Amiodarone and amlodipine would not have an effect on the supraventricular effect of the arrhythmia.

A 16-year-old high school boy presents to the emergency department 4 hours after sustaining an abrasion to his knee after a fall while rollerblading on the school playground. His school immunization record reveals that his last diphtheria, tetanus, and pertussis (DTaP) booster was administered at age 4. In this situation, which of the following is the MOST appropriate plan? A administer tetanus toxoid B administer adult tetanus and diphtheria toxoid (Td) C administer diphtheria, tetanus toxoid, and acellular pertussis (Tdap) vaccine D administer tetanus immune globulin

Administer diptheria, tetanus toxoid, and acellular pertussis (Tdap) vaccine Generalized tetanus (lockjaw) is a neurologic disease caused by Clostridium tetani. Although any open wound is a potential source for contamination with C tetani, those with dirt, soil, feces, or saliva are at increased risk. Tetanus-prone wounds contain devitalized tissue, especially those caused by punctures, frostbite, crush injury, or burns. Recommendations for tetanus prophylaxis in a child with a laceration or abrasion depend upon the number of previous vaccinations, occurrence of last booster, type of wound (clean or tetanus-prone), and age of child. In this case, the patient is older than 7 years and had all of his previous immunizations; however, his most recent booster was greater than 10 years ago. Thus, he should receive an adult-type diphtheria and tetanus toxoid with acellular pertussis. In most cases, when tetanus toxoid is required for wound prophylaxis in a child older than 7 years, the Td instead of tetanus toxoid alone is recommended so that diphtheria immunity is maintained. If tetanus immunization is not up to date at the time of wound treatment, then the immunization series should be completed according to the primary immunization schedule. If a child is younger than 7 years, then the diphtheria, tetanus, acellular pertussis (DTaP) booster is indicated, unless there is a contraindication for pertussis, in which case the diphtheria and tetanus (DT) booster should be administered. Tetanus immune globulin (TIG) is recommended for treatment of tetanus. Under special circumstances, a patient infected with the human immunodeficiency virus (HIV) with a tetanus-prone wound should also receive TIG in addition to the prophylactic vaccine.

What age group is most at risk to develop osteoid osteoma? A Adolescents B Elderly C Middle-aged D Neonates E Toddlers

Adolescents Osteoid osteoma is a benign bone forming tumor that usually develops during a patient's second decade of life. This type of tumor is much more common in boys than girls and typically affects the lower extremities (femur and tibia primarily) and spine more than other areas of the body. Patients typically present with gradually progressive bone pain that is worse at night and does not correlate with activity level. The tumor produces high levels of prostaglandins, so symptoms usually improve in 20-25 minutes if the patient takes a medication like ibuprofen, ASA or other NSAIDS that are prostaglandin inhibitors. A lack of improvement in symptoms with these medications should lead health care providers to consider a different diagnosis. The pain of this condition may cause those afflicted in a leg to limp and have swelling, muscle atrophy or contractures and exquisite point tenderness. The condition usually resolves on its own over time, but symptomatic patients may require surgical resection or radioablation of the tumor.

A 16-year-old girl is brought to the emergency department by ambulance after reportedly ingesting "a bottle of aspirin." Vital signs are temperature 37.8°C oral; pulse 94/min; respirations 30/min; blood pressure 100/68 mm Hg. What would you expect the blood gases to show that would confirm she had swallowed the aspirin? A anion gap metabolic acidosis with respiratory acidosis B nonanion gap metabolic acidosis with respiratory alkalosis C anion gap metabolic acidosis with respiratory alkalosis D nonanion gap metabolic acidosis with respiratory acidosis

Anion gap metabolic acidosis with respiratory alkalosis An acute salicylate overdose (greater than 150 mg/kg) will produce symptoms of salicylate intoxication. Chronic salicylate intoxication occurs with ingestion of greater than 100 mg/kg/day for at least 2 days. Salicylates affect most organ systems, leading to various metabolic abnormalities. Because salicylates are a gastric irritant, symptoms of vomiting and diarrhea occur soon after the overdose, which may contribute to the development of dehydration. Salicylates stimulate the respiratory center leading to hyperventilation and hyperpnea resulting in respiratory alkalosis and compensatory alkaluria. A characteristic feature of salicylate intoxication is the coexistence of a respiratory alkalosis with a widened anion gap metabolic acidosis.

A 20-year-old male presents to the emergency department complaining of pain to the right shoulder region while playing basketball. He states that his arm was pulled back and rotated while he was moving forward, and then felt a popping sensation in the shoulder. Since then he has not been able to move the shoulder at all due to pain and immobility. It is suspected that he has a dislocation. Given this scenario, what would be the most likely type? What type of shoulder dislocation did he most likely experience? A Anterior B Inferior C Multidirectional D Posterior E Superior

Anterior All of the various types of dislocations mentioned above are possible, but anterior dislocations are by far the most common (>95%) and they are the most common of all joint dislocations. Most occur as a result of a fall or other traumatic event and they may become recurrent. The shoulder is most susceptible to an anterior dislocation when it is abducted and externally rotated. The shoulder joint is considered a very mobile joint, but this also renders it very susceptible to injury.

A 54-year-old man with a history of chronic alcohol abuse presents to the emergency department with complaints of a subjective fever and severe epigastric pain radiating to the back. The pain has been present for the past 8 hours and is associated with nausea and vomiting, which has not relieved the pain. Laboratory data reveal a WBC of 14,000/mm 3 and a serum amylase of 500 U/L (reference range 0-286 U/L). Plain films of the abdomen were unremarkable. Which of the following is the most likely diagnosis? A Perforated duodenal ulcer B Acute cholecystitis C Acute pancreatitis D Mesenteric ischemia E Choledocholithiasis

acute pancreatitis C Acute pancreatitis typically presents with severe, steady midepigastric abdominal pain that radiates through to the back; pain is associated with fever, nausea, and vomiting. The most common causes of acute pancreatitis are gallstones and alcohol. Laboratory studies will show elevated WBC and serum amylase levels. Amylase elevations are nonspecific and can be elevated with perforated ulcers and mesenteric ischemia. A perforated ulcer will show evidence of free air on plain film; mesenteric ischemia will not present with fever or an elevated WBC unless there is the presence of infarcted bowel at which point the patient would appear septic. Acute cholecystitis may be associated with elevations in amylase but they are typically only a modest increase

A 24-year-old HIV-positive man comes to the emergency department complaining of severe left-sided chest discomfort, which radiates through to the left trapezius region. On coming into the room, you note that he is sitting up and hunched forward. On physical examination, the patient's blood pressure is 135/78, with a pulse of 85 bpm, and a pericardial friction rub is noted. Laboratory findings demonstrate elevated serum creatine kinase levels and normal serial troponin levels. His EKG demonstrates global ST segment elevation. His CXR demonstrates no acute process. Which of the following is the most likely diagnosis in this patient? A Acute pericarditis B Acute myocardial infarction C Acute bacterial endocarditis D Acute ascending aortic dissection E Acute costochondritis

acute pericarditis A Choice A is the most likely finding, as this patient is exhibiting signs, symptoms, and EKG findings pathognomonic for acute pericarditis, which is likely infectious in the setting of a patient with HIV. A pericardial friction rub is heard best with the patient in a seated position, during expiration, and is frequently found in patients with pericarditis. Choice B, an acute myocardial infarction, is less likely in a patient of this age, especially with normal serial troponins. Acute pericarditis can sometimes present with elevated serum creatine kinase levels when the epicardium is also involved. Choice C, acute bacterial endocarditis, is less likely in a patient with these EKG changes. Choice D, aortic dissection, would present with chest pain; however, the patient would be markedly hypotensive, less stable on presentation, and a CXR would demonstrate widening of the superior mediastinum

A 30-year-old man is brought to the emergency department by his wife for abdominal pain, nausea, vomiting, and diarrhea. She says he has been getting "worse and worse" for at least the past two months. He is "too tired" and his "muscles hurt too much" to play golf, formerly their favorite leisure activity. He has been depressed and reticent, instead of his usual talkative self. He won't eat much of anything, even when she cooks his favorite meals and she is sure he has lost considerable weight because his clothes "hang off him." He has refused medical evaluation until today, when the abdominal pain of approximately a week increased and he began having the vomiting and diarrhea. Initial impression is that the man is quite tanned, although it is the middle of winter. Closer examination reveals dark pigmentation in his skin folds and on the buccal mucosa. This presentation most strongly suggests which of the following? A adrenocortical insufficiency B diabetic ketoacidosis C hypercortisolism D hyperglycemic hyperosmolar state

adrenocortical insufficiency A This is a fairly classic presentation of adrenocortical insuffiency (Addison's disease). Diabetes mellitus, type 1, that has led to ketoacidosis (B) could also present with weight loss, nausea, and vomiting; but is also accompanied by hyperphagia and polydipsia, and does not have the hyperpigmentation associated with Addison's disease. Patients with hypercortisolism (Cushing Syndrome) (C) present with central obesity, hirsutism, thin skin, poor wound healing, and a host of other problems including emotional lability. Those with hyperglycemic hyperosmolar state (D) are typically older, have a high body mass index, and present with lethargy, confusion, and dehydration. Patients with hypothyroidism (E) usually have weight gain and constipation along with lethargy, fatigue, and weakness.

A 61-year-old man arrives at the emergency department (ED) suffering an acute myocardial infarction as a result of coronary artery thrombosis. One of the agents administered to the patient is a thrombolytic agent. From the choices below, which drug is a thrombolytic agent? A abciximab B alteplase C warfarin D heparin E clopidogrel

alteplase B Both warfarin and heparin are anticoagulants that are indicated for the prevention of thrombi. They do not actively lyse clots, but are capable of preventing further thrombogenesis. Both abciximab and clopidogrel are considered antiplatelet agents. Abciximab inhibits the activation of glycoprotein IIb/IIIa receptors on platelets, which helps to reduce platelet aggregation. Clopidogrel blocks adenosine diphosphate (ADP) receptors on platelets. The binding of ADP to these receptors is an important cellular mechanism in stimulating platelet aggregation. Alteplase converts plasminogen to plasmin, which then actively dissolves the fibrin threads associated with a thrombus.

A 36-year-old male is brought to the emergency department in handcuffs, after he sustained a laceration to his left leg while trying to outrun the police. Apparently, he was driving a stolen car when he was pulled over for speeding. Upon talking with him, you learn that he has been in jail twice before for robbery. During the history and physical exam he appears callous and impulsive. What is the most appropriate classification for this personality disorder? A Antisocial B Avoidant C Narcissistic D Paranoid E Schizoid

antisocial A Antisocial personality disorder consists of clinical findings that include selfishness, callousness, promiscuousness, impulsive behavior, and an inability to learn from experience and legal problems. Avoidant personality disorder presents clinically as someone who fears rejection, overreacts failure, has poor social endeavors, and low self-esteem. Narcissistic personality disorder presents with the clinical findings of grandiosity, a preoccupation with power, lacking interest in others, and excessive demands for attention. Clinical findings of someone who has paranoid personality disorder would include defensiveness, being overly sensitive, secretive, suspicious, and hyper-alert, and having a limited emotional response. Clinical findings of schizoid personality disorder include being shy, introverted, withdrawn, and avoiding close relationships.

A 55-year-old woman with a history of mitral stenosis, secondary to rheumatic heart disease, presents to the emergency department with increasing dyspnea while walking up one flight of stairs. She denies chest pain and discomfort, but states that recently she has also noticed palpitations. She also admits to lower extremity edema, which is new within the last week. Which of the following tachyarrhythmias is she most likely to demonstrate on EKG? A Ventricular tachycardia B Atrial flutter C Ventricular fibrillation D Ventricular bigeminy E Torsades de pointes

atrial flutter Choice B is correct. As patients with mitral stenosis age, and their mitral stenosis progresses to moderate or moderately severe mitral stenosis (most commonly after their fourth decade), the incidence of atrial arrhythmias—including premature atrial contractions, paroxysmal tachycardia, atrial flutter, and atrial fibrillation—increases. Choices A, C, D, and E are less likely, given that they are ventricular arrhythmias.

Shoulder dislocation commonly involves injury to which nerve? A Axillary B Median C Peroneal D Radial E Ulnar

axillary A The axillary nerve is in close proximity to the glenohumeral joint, thus making it vulnerable to injury during a shoulder dislocation. The median, radial, ulnar nerves are more distal in the upper extremity and and the peroneal nerve is in the leg.

n 18-year-old female comes to the emergency room stating she was raped just a few hours ago. She is not otherwise harmed, but does admit to a history of physical abuse between the ages of six and eight at the hands of a family member. Given this history, you know she is at higher than average risk for post-traumatic stress syndrome. You care for her immediate medical needs and are ready to release her. In counseling her on next steps, which of the following is most critical in order to improve her prognosis? A Begin therapy as soon as possible B Begin treatment with a serotonin uptake inhibitor C Begin treatment with a beta blocker to reduce symptoms D Begin treatment with prazosin to prevent sleep disturbance

begin therapy as soon as possible A Therapy to aid in working through the traumatic experience (A), instituted as soon as possible after the event, has proven to be the most helpful way to avert or minimize post-traumatic stress disorder. This patient does not currently have any symptoms of PTSD, so pharmacologic treatment is not indicated. In a patient who is diagnosed with PTSD, SSRIs (B) may be helpful in reducing panic and improving sleep, beta-blockers (C) may reduce symptoms of anxiety, and prazosin (D) may help with sleep. Legal procedures (E) may help a patient in dealing with the event, but there are no data supporting this as a therapeutic intervention.

A 48-year-old man presents to the emergency department with acute right upper quadrant tenderness, fever, and mild jaundice. Which of the following is most likely to be elevated in the blood? A bilirubin B creatinine C glucose D ketones E uric acid

bilirubin A This patient's signs and symptoms correlate with a suspected case of cholecystitis. Jaundice is associated with hyperbilirubinemia, in which the excess bilirubin can deposit in tissues such as the skin, sclera, and nails, causing a yellowish discoloration. Bilirubin is the waste product generated from the metabolism of hemoglobin.

You are doing your usual morning rounds with your supervising physician when you are called to the emergency department with a new admission. In room 7 of the emergency department you note an ill-appearing 72-year-old male with a diagnosis of acute renal failure due to postrenal azotemia. Which of the following would be the most likely cause of his postrenal azotemia? A renal artery stenosis B nonsteroidal anti-inflammatory drug (NSAID) use C massive bleeding D dehydration E bladder outlet obstruction

bladder outlet obstruction E The correct answer is (E). Postrenal azotemia is caused by an obstructive process of the bladder, urethra, or ureters. In this case the patient is likely to have bladder outlet obstruction as a cause of his postrenal azotemia. In older med with benign prostatic hypertrophy (BPH), the patient may present with n/v, abdominal pain, and bladder distension. Relief of the obstruction should relieve the patient's symptoms and potentially the acute renal failure. All other choices are suggestive of prerenal azotemia.

A 3 year-old boy is brought to the emergency department due to acute onset of cough and wheezing. Physical exam reveals focal wheezing in the right lower lobe. Which of the following is the most effective treatment option for the patient's suspected diagnosis? A Albuterol B Azithromycin C Bronchoscopy D Chest physiotherapy E Prednisone

bronchoscopy The patient has most likely aspirated a foreign body. The most effective treatment is removal of the foreign body through bronchoscopy (C). Supportive therapy includes bronchodilators for wheezing/airway obstruction (A), antibiotics for associated infections (B), and chest physiotherapy (D) to promote recovery after removal of the foreign body.

A 78-year-old woman with a medical history of diabetes and hypertension presents to the emergency department complaining of left hand weakness and slurred speech. Which of the following tests is most likely to determine the source of an arterial thrombus? A carotid ultrasound B CT of the brain C erythrocyte sedimentation rate D magnetic resonance angiography (MRA) of the vertebral arteries

carotid u/s The patient's symptoms are consistent with pathology arising from the anterior cerebral circulation including the carotid arteries. A CT should be ordered to rule out acute hemorrhage and an erythrocyte sedimentation rate may be useful if giant cell arteritis were suspected. An MRA of the vertebral arteries would likely show deficits but is not likely to demonstrate the etiologic location of this stroke.

A 62-year-old female with a known history of hypertension presents due to the abrupt onset of a severe headache, marked vertigo, nausea, vomiting, and ataxia. On physical exam, you note nystagmus, sensory loss, and weakness of the right face. What is the most likely diagnosis? A Cervical artery dissection B Cerebellar hemorrhage C Internal auditory artery occlusion D Lacunar infarction E Spinal cord compression

cerebellar hemorrhage A cerebellar hemorrhage can result from multiple causes, similar to other intracerebral hemorrhages, such as hypertension, blood dyscrasias, trauma, and arteriovenous malformations. Patient presentation may range from an abrupt onset of headache to full coma, depending on the severity of the hemorrhage. Presentation is often similar to cerebellar infarction, with CT findings assisting in the differentiation. Symptoms may include all of those listed above as well as nuchal pain, altered consciousness, altered respiratory rate, abnormal eye movements, and impaired papillary responses. Cervical artery dissection may present with similar symptoms, including the abrupt onset of headache, but is not typically associated with ataxia, vertigo, or vomiting. Internal auditory artery occlusion is associated with vertigo and unilateral hearing loss. Lacunar infarction occurs within small resistance vasculature, with motor and sensory symptoms related to the impacted brain region. Spinal cord compression may be acute or progressive, with motor weakness and sensory loss caudad to the lesion.

An 8-year-old girl is brought in to the emergency department with abdominal cramps, nausea, and vomiting since early this morning. She has had two loose stools but denies melena or hematochezia. She has had a low-grade fever. In the past hour, her vision has become blurry and she feels increasingly weak. Her mother has had similar but milder symptoms. Twenty-four hour dietary recall includes only chicken broth today. Last night for dinner they had meatloaf (fully cooked), mashed potatoes, and green beans. Her mother cans all their vegetables. Her medical history is unremarkable. She takes no medications. No known drug allergies. Examination reveals a temperature of 99°F, clear lungs, and mildly tachycardic heart with no murmur audible. Abdomen-bowel sounds present, soft with mild diffuse tenderness, no guarding. Neurologic examination is significant for decreased visual acuity and decreased motor strength (2/5) in the upper and lower extremities. The most likely etiology is A enterotoxic E coli B cholera species C pinworms D Clostridium botulinum

clostridium botulinum Clostridium botulinum produces a neurotoxin that can lead to life-threatening illness including respiratory paralysis. Botulism infection is caused by the spore-forming bacteria that lives in soil and can be foodborne. In the latter case, home-canned foods are often the cause. After a 12-hour to 3-day incubation period, botulism begins with classic symptoms of abdominal pain, nausea, vomiting, and mild diarrhea and, if unchecked, evolves into a progressive neurologic disorder marked by double vision, motor weakness, and ptosis. Respiratory muscle involvement may occur ultimately and result in death. Because of the virulence of the neurotoxin it has been used as an agent of bioterrorism. Cholera and enterotoxigenic E. coli cause a foodborne diarrheal illness that can result in significant morbidity and mortality, but they do not have neurologic manifestations. Pinworm infection is usually found among younger children, is marked by severe anal itching, and fecal-oral transmission

A patient who is intoxicated presents to the emergency department. On ocular exam, you notice mydriasis. Which substance could he have been using? A sedatives B PCP C opioids D cocaine

cocaine Patients intoxicated with cocaine present with mydriasis. In opioid intoxication, the pupils are constricted. PCP intoxication is associated with nystagmus.

A 17-year-old male is brought to your Emergency Department by his girlfriend. She states that he has been behaving strangely for the last three days, with rapidly fluctuating moods ranging from euphoric to irritable and paranoid. The patient states that he is fine, just a little nervous about an upcoming test in school. His pulse is 126 beats per minute, BP 182/106, pupils are widely dilated, and he is diaphoretic. What is his most likely diagnosis? A Acute anxiety B Bipolar disorder C Cocaine intoxication D Heroin intoxication E Thyrotoxicosis

cocaine intoxication This patient's presentation with tachycardia, hypertension, diaphoresis, and mydriasis along with the behavioral changes is consistent with cocaine intoxication (C). His physical symptoms could be explained by thyrotoxicosis (E), but he has no history of hyperthyroid symptoms such as weight loss and it would not explain the mydriasis. Acute anxiety (A) or bipolar disorder (B) might explain some of the mood changes but would not account for all of his physical signs. Heroin withdrawal (D) would cause some similar symptoms, but intoxication would cause somnolence and pinpoint pupils.

A 24-year-old man with a recent history of a viral illness comes to the emergency room complaining of severe left-sided chest discomfort, which radiates through to the left trapezius region. On coming into the room, you note that he is sitting up and hunched forward. On physical examination, the patient's temperature is 39°C, blood pressure is 135/78, with a pulse of 85 bpm, and a pericardial friction rub is noted. Laboratory findings demonstrate elevated serum creatine kinase levels and normal serial troponin levels. Which of the following would be the most likely electrocardiographic findings? A Diffuse ST segment elevation B Peaked T waves C Inferior Q waves D Loss of R-wave amplitude E U waves

diffuse ST segment elevations In a patient with these signs, symptoms, and lab findings, acute pericarditis is the most likely diagnosis. In patients with acute pericarditis, EKG changes occur secondary to inflammation of the subepicardium, leading to widespread elevation of the ST segments, often with upward concavity, which returns to normal after several days, followed by T wave inversion. No significant QRS complex changes are noted, so choice C, the development of inferior Q waves (frequently associated with an inferior myocardial infarction), is incorrect. Choice B is frequently noted with severe hyperkalemia. Loss of R-wave amplitude, choice D, is associated with myocardial infarction. Choice E, U waves, are associated with hypokalemia.

A 66-year-old male with a history of hypertension, diabetes mellitus, and hypercholesterolemia presents by emergency medical services (EMS) to the emergency department complaining of severe chest pain with radiation into his back. The patient states that he was feeling well in the morning, but while performing some light activity he felt a "ripping" sensation in his back, which he initially thought was a pulled muscle. The pain continued and the patient started to have chest pain, shortness of breath, and lightheadedness. On initial examination the patient is still in pain, pale, diaphoretic, and has a blood pressure of 85/40. His chest is clear to auscultation, and he has a 3/6 diastolic murmur best appreciated at the base of the heart. Given this clinical scenario, what is the most likely diagnosis? A Pneumothorax B Dissecting thoracic aneurysm C Acute myocardial infarction D Pulmonary embolus

dissecting thoracic aneurysm B This patient is exhibiting a history and physical examination that is consistent with a thoracic aneurysm. The patient's history of hypertension, along with the "ripping" sensation in his back and hypotension give a clinical presentation that is suggestive of a thoracic aneurysm dissection (B). A pneumothorax (A) would have more pleuritic characteristics, and chest pains without the ripping sensation or loud diastolic murmur would be more likely in a situation of myocardial infarction (C). Pulmonary embolus (D) and esophageal perforation (E) would typically not present with the above complaints or physical exam findings.

A 14-year-old boy presents to the emergency department with acute scrotal pain and vomiting for the past 2 hours. His left testicle is in extreme pain and he states the pain started while playing basketball in gym class. Which diagnostic test would help confirm your suspected diagnosis? A Transillumination B Doppler ultrasound C Urine cultures and sensitivity D Radionuclide imaging E Serum human chorionic gonadotropin levels

doppler u/s B Doppler ultrasound (B) is the diagnostic imaging of choice to confirm testicular torsion, with radionuclide imaging (D) a distant second. Testicular torsion is time sensitive and MRI or more invasive imaging may lead to the delay of surgical intervention. Time is critical (< 6 hours) for the salvage of the affected testicle. Transillumination (A) is seen in hydrocele fluid. Urine cultures and sensitivity (C) can confirm urinary tract infections or sexually transmitted diseases causing epididymitis. If an intrascrotal tumor is suspected, serum tumor beta-human chorionic gonadotropin levels (E) should be obtained.

In the emergency department, you are asked to evaluate a 77-year-old man with a history of HTN who had a syncopal episode while chasing after his dog. He admits to recent episodes of chest discomfort, also associated with activity, as well as dyspnea at lower levels of activity including walking up one flight of stairs. On physical exam, a grade III/IV crescendo-decrescendo systolic ejection murmur can be heard best over the right upper sternal border. His EKG demonstrates NSR @ 80 bpm, with evidence of left ventricular hypertrophy. His troponin levels are negative for ischemia. What is the next most appropriate test or procedure? A Echocardiography B VQ scan C CT scan of the head D Serum D-dimer levels

echo A This patient exhibits all the signs of progression of aortic stenosis, thus echocardiography is the next most appropriate test. A determination of severity can then be made, with possible cardiac catheterization if severe aortic stenosis is suspected, in preparation for surgical intervention if necessary. A VQ scan is appropriate if pulmonary embolism were suspected. A CT scan of the head could be considered if a head injury was suspected, but would not be the next step in the management of this patient. Serum D-dimer levels might be used to rule out pulmonary embolism, although it is a fairly nonspecific test. An MRI of the heart is not considered standard of care for aortic stenosis

What is the most common joint dislocation in children? A Ankle B Elbow C Finger D Knee E Shoulder

elbow In children the elbow is the most commonly dislocated joint and it is the third most common joint dislocation for adults. The shoulder and finger are dislocated more frequently than the elbow in adults. Knee dislocations at the femur-tibia joint are rare (patellofemoral dislocations are more common) and ankle dislocations are also relatively rare

A 55-year-old woman with a history of emphysema, who is undergoing chemotherapy for lung cancer, comes to the emergency department complaining of a sudden increase in dyspnea, with exertion and fatigue. On physical exam, hypotension, pulsus paradoxus, and muffled heart sounds are noted. On transthoracic echocardiography, cardiac tamponade is noted, with over 200 mL of pericardial fluid described. Which of the following would be this patient's most likely electrocardiographic finding (Figure 7)? A Torsades de pointes B U waves C Electrical alternans with sinus tachycardia D Peaked T waves E Convex elevation of the J point

electrical alternans C Choice C, electrical alternans with sinus tachycardia, a beat-to-beat alteration in one or more components of the ECG signal, is considered a specific sign of pericardial effusion, often with cardiac tamponade, as it represents the periodic swinging motion of the heart in the effusion at a frequency that is ½ the heart rate. Choice A, torsades de pointes, is a type of ventricular tachycardia frequently seen, and is associated with electrolyte disturbances or the use of certain types of antiarrhythmic drugs. Choice B, U waves, are associated with hypokalemia. Choice D is frequently noted with severe hyperkalemia. Choice E, convex elevation of the J point, is seen in patients suffering from hypothermia

A 55-year-old woman with a history of emphysema, who is undergoing chemotherapy for lung cancer, comes to the emergency room complaining of a sudden increase in dyspnea, with exertion and fatigue. On physical exam, hypotension, pulsus paradoxus, and muffled heart sounds are noted. On transthoracic echocardiography, cardiac tamponade is noted with over 200 mL of pericardial fluid described. Which of the following is the most appropriate next step in management? A Enoxaparin 1 mg/kg subcutaneously Q 12 H B EKG C Cardiac catheterization D Penicillin V 500 mg PO BID x 10 days E Emergent pericardiocentesis

emergent pericardiocentesis E Choice E, emergent pericardiocentesis, should be considered when patients exhibit symptoms suggestive of severe cardiac tamponade, such as described above, with confirmatory echocardiographic findings of a large pericardial effusion, as this can be fatal if not treated promptly. Choice A is inappropriate, as this could lead to worsening of cardiac tamponade if it is secondary to bleeding into the pericardial space, such as with trauma or postoperatively. Choice B could provide additional confirmation of the diagnosis if a reduction in amplitude of QRS complexes is demonstrated, and electrical alternans of the P, QRS, and T waves, but would not provide any therapeutic benefit. Choice C could be the next appropriate step in management of myocardial infarction, but not in cardiac tamponade. Choice D represents appropriate therapy for patients with acute rheumatic fever, but not for patients with cardiac tamponade.

A 67-year-old female presents to the emergency department with abdominal pain, bloating, inability to pass gas or stool, and vomiting. Which of the following, if present, would be a contraindication to placing a nasogastric tube? A Esophagitis B Esophageal strictures C Esophageal varices D Peptic ulcer disease E Gastroesophageal refulx disease

esophageal strictures Contraindications to nasogastric tube placement include choanal atresia, significant facial trauma, basilar skull fracture, esophageal stricture or atresia, esophageal burn, zenker's diverticulum, recent surgery on the esophagus or stomach, or a history of gastrectomy or bariatric surgery.

A 39-year-old man presents to the emergency department with massive hematemesis. His physical examination reveals slight jaundice, palmar erythema, spider angiomas, and marked ascites. Vitals at the time of presentation are as follows: BP: 85/44 mm Hg, P: 122/min, R: 16/min, oxygen saturation: 96%, and T: 99.8°F. Which of the following is the most likely cause of the massive hematemesis? A Peptic ulcer disease B Mallory-Weiss tear C Gastric carcinoma D Arteriovenous malformation E Esophageal varices

esophageal varices The most common cause of massive upper gastrointestinal bleeding in patients with cirrhosis is esophageal varices. Although 20% of patients with portal hypertension will have bleeding from other causes (peptic ulcer disease, Mallory-Weiss tears, or gastritis), endoscopic evaluation in patients with portal hypertension is necessary for diagnosis and initial therapy.

A 20-month-old boy is brought into the emergency department by his parents. They state he has not been feeling well for 2 days and this morning noted he was "shaking all over" and was not responding to commands. This went on for less than 10 minutes and has never happened before. His current rectal temperature is 100.7°F. The seizures are characteristic of A absence seizures B Lennox-Gastaut syndrome C febrile seizures D infantile spasms E juvenile myoclonic epilepsy

febrile sz Febrile seizures can occur in children younger than 5 years when accompanied by a fever. They are characterized by a brief generalized motor seizure. Absence seizures are generalized seizures characterized by a loss of consciousness without motor involvement, typically seen in older children. Lenox-Gastaut syndrome presents in childhood as well but is usually associated with developmental delay and seizures of akinetic and myoclonic nature (referred to as drop attacks). Infantile spasms occur without relation to systemic illness and are massive myoclonic events with bending at the waist. Juvenile myoclonic epilepsy evolves in the teenage years and is characterized by repeated episodes of myoclonic seizure activity.

A 88-year-old female is found lying on the ground in a supine position after sustaining a fall in her house from tripping on the rug.. Her leg is shortened, abducted and externally rotated, and she is complaining of pain to the right leg with radiation to the knee. Based on the history of the patient, what type of injury has this patient sustained? A Non-displaced pelvic fracture B Hip dislocation C Femoral shaft fracture D Femoral neck fracture E Distal femur fracture

femoral neck fracture When a fracture occurs in the femoral neck that includes displacement, the leg appears shortened, abducted and externally rotated. The tension that exists from muscles attached above and below the fracture site results in shortening of leg length after the fracture as the muscles contract. If there is no displacement of the bone, no leg length changes should occur. Generally, hip fractures would not result in a lengthening of the leg, so all answers that include this option would be wrong. When the leg appears shortened, flexed, adducted, and internally rotated, a posterior hip dislocation should be suspected. A leg that is flexed, abducted and externally rotated suggests an anterior hip dislocation. Posterior hip dislocations are far more common than anterior dislocations, accounting for more than 90% of all hip dislocations.

A 4 year-old male presents to the emergency department with vomiting, severe abdominal pain, and diarrhea that occurred 2 hours after "getting candy from Grandma's purse". Which of the following medications did the child most likely ingest? A Cisplatin B Erythropoietin C Ferrous sulfate D Lisinopril E Vincristine

ferrous sulfate Stage 1 of iron toxicity secondary to ferrous sulfate ingestion (C) is characterized by acute GI irritation; this is followed by a latent phase (Stage 2) that can then progress to systemic iron toxicity (Stage 3), that can progress to hepatic failure (Stage 4) or delayed sequelae (Stage 5). Cisplatin (A), erythropoietin (B), and vincristine (E) are parenterally administered and unlikely to be present in a patient's purse. Lisinopril (D) toxicity consists of cardiovascular symptomatology (e.g., hypotension and tachycardia).

In a boxer's fracture, the presence of how much angulation of the fifth metacarpal neck would require referral for reduction as opposed to treatment with a simple ulnar gutter splint? A Less than 10 degrees B 10-19 degrees C 20-29 degrees D 30-39 degrees E Greater than 40 degrees

greater than 40 degrees When there is greater than 40 degrees of angulation or an extensor lag (the patient cannot fully extend the affected finger) then a referral is required. Lesser degrees of angulation without an extensor lag can generally be handled conservatively with an ulnar gutter

A 25-year-old male has a history of schizophrenia, and was brought to the emergency department by ambulance after he was found wandering along the highway. Which of the following is considered a positive symptom of schizophrenia? A Catatonia B Diminished sociability C Hallucinations D Poverty of speech E Restricted affect

hallucinations C Hallucinations, delusions, and formal thought disorders are classified as positive symptoms of schizophrenia. Negative symptoms of schizophrenia include restricted affect, diminished sociability, and poverty of speech. Catatonia describes a state in which a person is withdrawn and shows minimal bodily movement, or one in which there is severe excitement and the person shows purposeless and stereotyped movements.

A 64-year-old man has been experiencing signs and symptoms compatible with diverticular disease for the past 3 weeks. He now presents to the emergency department malnourished with severe left-sided lower abdominal pain. After appropriate workup and hydration, he is taken to the operating room where a perforated sigmoid colon is discovered with gross contamination. What is the most appropriate surgical intervention at this time? A Left colectomy with primary anastomosis B Hartmann procedure C Proctocolectomy D Abdominoperineal resection E Low anterior resection

hartmann procedure This vignette is consistent with an emergent resection in an unprepared patient. The most appropriate therapy for an acute perforation is a Hartmann procedure, which includes resection of the affected portion of the bowel, a temporary diverting colostomy, and oversewing of the distal rectal stump; the second stage of the procedure will involve taking down the colostomy with anastomosis to the rectal stump. A colectomy with a primary anastomosis should not be done when the bowel is unprepared due to the significant risk of infection and leakage of the bowel at the site of the anastomosis. Abdominoperineal resection is used in the treatment of malignant disease of the lower rectum. In this procedure, a permanent colostomy is created and the entire rectum, anal canal, and anus are removed. In the management of benign disease of the lower rectum, a proctocolectomy is appropriate to preserve anal function.

A 76-year-old man with a history of three myocardial infarctions, CABG x 4, COPD, HTN, and hyperlipidemia presents to the emergency department with complaints of increasing dyspnea with exertion. He now has to sit in his recliner to sleep. He complains of lower extremity edema, now above the knees. On physical examination, he exhibits jugular venous distention to the angle of the jaw at 90 degrees of truncal elevation, diffuse bilateral rales on auscultation of the lung, and a blowing systolic murmur that is heard best at the left lower sternal border. Which abdominal physical exam finding would be most likely given this patient's history? A Periumbilical ecchymoses B Murphy's sign C Tenderness at McBurney's point D Hepatomegaly E Absent bowel sounds

hepatomegaly D Choice D is correct, as the patient is demonstrating signs and symptoms of congestive heart failure, which can lead to hepatic congestion and hepatomegaly. Choice A, periumbilical ecchymoses, or Cullen's sign, are representative of hemoperitoneum. Choice B, Murphy's sign, is indicative of cholecystitis. Choice C, tenderness at McBurney's point, is frequently seen with appendicitis. Choice E, absent bowel sounds, is seen in patients with ileus.

A 76-year-old man, is brought to the emergency department by his niece after she found him wandering around his yard in the cold wearing only a tee shirt and jeans. When she set up his pill container about 36 hours earlier, he seemed his usual self but, in retrospect, possibly a little more confused than usual. The niece says that he has "high blood," treated with a "white fluid pill," "sugar diabetes," treated with an oral medication, and early "old timer's" dementia treated with "a memory pill." Vital signs include an oral temperature of 100.8F, pulse 100 beats per minute, respirations 24 and somewhat shallow, and blood pressure of 88/52. Initial examination reveals a slightly dehydrated, stuporous man appearing older than his stated age, who smells strongly of urine. He has no lateralizing signs. What is the most likely cause of the mental status changes? A hyperglycemic hyperosmolar state B lactic acidosis C stroke D urinary tract infection E worsening dementia

hyperglycemic hyperosmolar state The combination of confusion and dehydration in a patient with diabetes type 2 who is taking a diuretic strongly suggest hyperosmolar state. Patients with lactic acidosis (B) have marked hyperventilation and, usually, signs and symptoms of a serious illness. The lack of lateralizing signs makes a stroke (C) less likely. Urinary tract infection (D) could certainly cause confusion and incontinence in an elderly man and should be investigated. Alzheimer dementia (E) progresses slowly; sudden decompensation is usually due to delirium.

A 63-year-old male with chronic kidney disease is in the ICU with bradycardia, flaccid paralysis, and an ileus. A stat work-up includes an ECG showing peaked T waves. Which lab finding would you suspect? A Hypomagnesemia B Hypercalcemia C Hypokalemia D Hypocalcemia E Hyperkalemia

hyperkalemia E Hyperkalemia (E), >5.0 mEq/L, usually occurs in patients with advanced kidney disease and impairs neuromuscular transmission including muscle weakness, flaccid paralysis, and ileus. ECG includes bradycardia, prolonged PR interval, peaked T waves, and QRS widening. Hypokalemia (C) can also present with flaccid paralysis or ileus but ECG findings show decreased and broadening T waves with prominent U waves. Hypomagnesemia (A) and hypocalcemia (D) have common symptoms of weakness and muscle cramps and can show prolonged QT intervals of ECG. Hypercalcemia (B) is often asymptomatic until >12 mg/dL, which includes constipation, polyuria, and nausea/vomiting, with ECG showing a shortened QT interval.

A 15-year-old boy suddenly collapses on the basketball court; his sports physical conducted at the beginning of the year did not elicit any abnormal findings. Basic life support initiated at the scene, however, is unsuccessful in resuscitation. Which of the following is the most likely etiology of his sudden death? A mitral valve prolapse B surgically corrected aortic stenosis C hypertrophic cardiomyopathy D rheumatic heart disease

hypertrophic cardiomyopathy C Hypertrophic cardiomyopathy in adolescence is typically due to familial hypertrophic cardiomyopathy with an incidence of 1:500. Many patients are asymptomatic until a sporting event, which may cause symptoms, specifically sudden cardiac death. Examination may demonstrate a palpable or audible S 4 , an LV (left ventricular) heave, systolic ejection murmur (may need to stimulate cardiac activity), and/or a left precordial bulge. Echocardiography is the gold standard for diagnosis but family history should be assessed. Stress testing is indicated to assess for ischemia and arrhythmias. Strenuous activities are prohibited for these patients. The other cardiomyopathies (dilated and restrictive) are next but are not as common. Congenital structural abnormalities of the coronary arteries are the next most common cause. Valvular disorders, including surgically repaired aortic stenosis, are typically not causes of sudden death, but these patients should be screened for symptoms and stress tested as necessary.

What is the most common clinical finding on vital signs in a patient with cardiogenic shock? A Fever B Hypertension C Hypotension D Tachypnea E Bradycardia

hypotension Hypoperfusion to the body is what generally causes the end organ failure associated with shock. The patient may also be tachycardic, but may not have fever or an increased respiratory rate as part of the abnormality. In most cases patients will have tachycardia unless they suffer a significant pump failure, which may cause bradycardia.

A 2-year-old baby girl is brought to the ED with a history of abdominal pain and diarrhea. Mother states that the child was playing normally and then "doubled over" with what appears to be abdominal pain. The abdomen appears slightly distended and is tender to palpation. While in the ED the child has a bloody, diarrheal bowel movement. Which of the following is the most likely diagnosis? A pyloric stenosis B mesenteric ischemia C Crohn disease D intussusception E Hirschsprung disease

intussusception D Intussusception is the most frequent cause of intestinal obstruction in the first 2 years of life. The patient develops paroxysms of pain followed by bloody bowel movements. Pyloric stenosis typically presents prior to the age of 6 months with vomiting but not with diarrhea. Hirschsprung disease results from an absence of ganglion cells in the colon and typically presents early in life with failure to pass meconium, followed by vomiting and abdominal distension. The typical age of onset is later in adolescence in Crohn disease and in the elderly in mesenteric ischemia.

An avulsion fracture at the base of the fifth metatarsal is commonly called which of the following? A Bennett fracture B Boxer's fracture C Chauffer's fracture D Jones fracture E Lisfranc fracture

jones fracture An avulsion fracture at the base of the fifth metatarsal, usually secondary to plantar flexion and inversion is called a Jones fracture. Also called a ballet or dancer's fracture, it is the most common metatarsal fracture. The fracture occurs at the proximal diaphysis. A Bennett fracture is an oblique fracture of the first metacarpal near the carpometacarpal joint. A boxer's fracture is a fracture of the fifth metacarpal. This is the most common fracture of the hand. A chauffer's fracture is an oblique fracture through the base of the radial styloid in the forearm. A Lisfranc fracture is actually a fracture and dislocation involving the tarsometatarsal joints

What types of connective tissue are injured in a sprain? A Bones and muscles B Fascia and joint capsules C Ligaments and joint capsules D Muscles and tendons E Tendons and bones

ligaments and capsules A sprain involves injury to those tissues that give support to joints - ligaments and joint capsules. Injury to muscles, tendons, and fascia would all be classified as a strain. Injuries to bone would be classified as a fracture.

A 35-year-old man is brought to the emergency department with unremitting, generalized convulsive status epilepticus. The initial, preferred treatment is intravenous administration of which of the following? A phenobarbital B valproate C phenytoin D lorazepam E donepezil

lorazepam D In most patients suffering from generalized convulsive status epilepticus (GCSE), benzodiazepines such as lorazepam and diazepam are effective initial therapies due to their relatively high lipid solubility. As a result, they are able to cross the blood-brain barrier easily, which gives them the potential to stop seizures quickly. Lorazepam's lipid solubility is less compared to diazepam, and it also redistributes to fat more slowly. Hence, lorazepam tends to have a longer duration of action (12 to 24 hours) than diazepam (20 to 30 minutes). Phenytoin is often administered immediately after benzodiazepine administration for long-term seizure control, as it has a long half-life (20 to 36 hours) compared to diazepam. Phenytoin is not given first because its lipid solubility is less than the benzodiazepines and therefore cannot enter the brain quickly enough to terminate seizure activity.

A 21-year-old male presents to the emergency department due to vomiting blood. He is a college student and was initiated into a fraternity earlier in the week, and admits to drinking enough alcohol to make him vomit that night. There was no blood in the vomit at that time. The hematemesis started today, about 20 minutes after he finished a workout at the gym. He notes that today was a particularly tough workout, as he increased the amount of weight he was lifting significantly compared to previous workouts. His vital signs are as follows: heart rate 86 bpm, respirations 14, BP 126/82, temp 98.8°F. What is his most likely diagnosis? A Mallory-Weiss syndrome B Peptic ulcer disease C Arteriovenous malformation D Esophageal varices E Erosive gastritis

mallory weiss syndrome A The correct answer is Mallory-Weiss syndrome. This is characterized by a nonpenetrating mucosal tear at the gastroesophageal junction, which may arise from a sudden increase in transabdominal pressure, such as lifting, retching, or vomiting. Peptic ulcer disease usually presents with a history of dyspepsia. Arteriovenous malformations usually present with melena or hematochezia. Esophageal varices usually develop secondary to portal hypertension, and are most often found in patients with cirrhosis. Erosive gastritis is often due to NSAIDs, alcohol, or severe medical or surgical illness, and is more often associated with chronic blood loss.

An elderly patient is brought in to the emergency department (ED) complaining of incontinence of liquid "like tea water" stool. He is complaining of rectal pressure and lower abdominal pain. The pain is cramping in quality and the patient's abdomen is "bloated." Digital rectal exam reveals hard stool in the rectum. Which of the following should be selected as the initial treatment for this patient? A passing a nasogastric tube B milk of magnesia C opiate analgesics for pain D oral sodium phosphate E manual disimpaction

manual disimpaction Mechanical bowel obstruction in the rectum does not usually respond to oral laxatives. A nasogastric tube would not be used for an obstruction in the distal colon/rectum. One would avoid opiates in fecal impactions and other constipation problems because they tend to be more constipating. This patient needs to be disimpacted. Oral agents are unlikely to be effective against the fecal impaction and may cause complications.

What is the most common radiologic finding in a patient with a tension pneumothorax? A Pleural effusion B Infiltrate C Enlarged cardiac silhouette D Elevated diaphragm E Mediastinal shift

mediastinal shift E Collapse of the lung, followed by an increase in intrapleural pressure, will lead to a tension pneumothorax. In most cases, the air leaks through the subcutaneous spaces. But in a tension pneumothorax, the air is locked in and creates a life-threatening emergency.

Your patient is a 14-year-old female, 5'4" weighing 80 pounds. Her menstrual cycle started at age 10 but stopped one year ago when she became concerned about her weight and began focusing on diet and exercise. Her mother brings her to the emergency room, stating she will only eat salads and runs 2 hours daily. What is the first consideration in the management of this patient? A Behavioral modification B Family therapy C Individual psychotherapy D Nutritional restoration E Pharmacologic intervention

nutritional restoration Nutritional deficiencies (D) such as dehydration and electrolyte imbalances must be urgently corrected. After metabolic imbalances have been addressed, a comprehensive, inter-disciplinary approach is optimal, with the primary goal of weight restoration. Behavioral, family, and individual therapies (A, B, and C) should all be a part of the longer-term management plan, and pharmacological therapy (E) may also be helpful.

A 16-year-old male was hit on the left side of his face by a line drive baseball. Marked swelling is noted externally to the left eye. There was no loss of consciousness. Upon physical exam, he complains of diplopia during extraocular motion testing. Enophthalmos is noted, as well as decreased sensation of the left cheek. Plain x-rays of the face demonstrate an air-fluid level in the left maxillary sinus, and a fracture of the orbit. Based on this information, what is the most likely diagnosis? A Zygomatic arch fracture B Orbital blowout fracture C Le Fort I fracture D Le Fort II fracture E Le Fort III fracture

orbital blow out fracture B Diplopia is common in an orbital blow out fracture, due to entrapment of the inferior rectus and inferior oblique muscles. Loss of infraorbital sensation occurs from disruption or swelling of the infraorbital nerve. A Le Fort I fracture describes a transverse fracture separating the body of the maxilla from the pterygoid plate and nasal septum. A Le Fort II fracture describes a pyramidal through the central maxilla and hard palate. Movement of the hard palate and nose occurs, but not the eyes. A Le Fort III fracture describes a craniofacial disjunction, wherein the entire face is separated from the skull due to fractures of the frontozygomatic suture line, across the orbit and through the base of the nose, and ethmoids. The entire face shifts, with the globes held in place only by the optic nerve.

A 33-year-old IV drug user presents to the emergency department with chills, diaphoresis, anorexia, and malaise. On physical exam, her temperature is 40°C, BP 98/55, P 115 bpm, and RR 22. Two separate blood cultures are positive for S.aureus. Which of the following physical exam findings would confirm a clinical diagnosis of infective endocarditis, according to the Duke criteria? A Increase in valvular regurgitation B Irregularly irregularly pulse C Osler's nodes D Buccal hemmorhages E Koplik spots

osler's nodes Choice C, Osler's nodes, confirms the clinical diagnosis of infective endocarditis, as it is a minor criteria. The Duke criteria for the clinical diagnosis of infective endocarditis requires the documentation of two major criteria, or one major criteria and three minor criteria, or five minor criteria. The patient demonstrates the presence of one major criteria (two separate blood cultures with typical microorganisms for infective endocarditis) and two minor criteria (fever greater than 38.0°C and predisposing condition of IV drug use). Only a new valvular regurgitation, not an increase or change in preexisting murmur, is considered sufficient to qualify as a major criteria, so choice A is incorrect. An irregularly irregular pulse, choice B, is commonly seen in patients with atrial fibrillation, not with infective endocarditis. Choice D, conjunctival hemorrhages, not buccal hemorrhages, are one of the minor criteria. Choice E, Koplik spots, are buccal lesions seen in patients infected with measles, whereas the presence of Roth's spots does fulfill one of the minor criteria.

Which of the following is a type of primary malignant tumor of the bone? A Enchondroma B Leiomyoma C Lipoma D Melanoma E Osteosarcoma

osteosarcoma E Osteosarcoma is a type of primary bone cancer. Fortunately this is a rare cancer, accounting for only 1% of the cancers diagnosed each year in the USA for patients of all ages. Osteosarcomas are more common in the pediatric population, accounting for 5% of all childhood cancers and 56% of all cancers of the bone in patients less than 20 years old. Osteosarcomas have a bimodal occurrence rate with the highest prevalence between ages 11-13 and over the age of 65. Enchondromas are common benign cartilaginous tumors that develop in the medulla (marrow cavity) of bone. A leiomyoma is another name for a benign uterine fibroid. A lipoma is a benign, soft, freely movable, generally nontender mass in the soft tissue sometimes referred to as a fatty tumor. These are generally inconsequential, but may be a marker for spina bifida if found in the lumbar region. Melanoma can be found in bone, but it would be considered a metastatic lesion and not a primary malignant bone tumor.

You are evaluating a 72-year-old male who is referred to the emergency department from his primary care office. On examination you note a lethargic obese male with the following vitals: BP of 225/135 a P of 88, and T of 98.4˚F. He reports discontinuing all his medications for blood pressure 6 months ago due to financial reasons. Which of the following physical examination findings would be most consistent with a diagnosis of a hypertensive emergency? A normal exam B xanthelasma C varicose veins D papilledema

papilledema D The correct answer is (D). This patient has significantly elevated blood pressure, lethargy, and evidence of hypertensive encephalopathy, which is supported by a physical examination finding of papilledema. Additional workup may be indicated to rule out other causes of his papilledema. Target organ damage is apparent in anyone with hypertensive emergencies and may be neurological, renovascular, cardiovascular, or a combination thereof. Prompt treatment is necessary to prevent complications. Although the physical examination may be normal, evidence of end-organ damage is likely on examination and/or laboratory/diagnostic studies. Choices (B), (C), (E) are not representative of end-organ damage seen in hypertensive emergencies.

A 17-year-old female distance runner with no significant PMH complains that she has diffuse, aching anterior knee pain that is worsened when she walks up or down stairs or when she squats down. There has been no acute trauma, but she has been increasing her running mileage. No effusion is present. What is the probable diagnosis? A Anterior cruciate ligament tear B Medial meniscal tear C Osteoarthritis of the knee joint D Patellofemoral syndrome

patellofemoral syndrome D Patellofemoral syndrome is a common condition in active adolescents due to repetitive stresses on the patellofemoral joint. This can be exacerbated by altered patellofemoral tracking due to growth and development in adolescents. Other biomechanical issues that may contribute to this problem include poor flexibility, weakness of the vastus medialis muscle, which results in a more lateral tracking of the patella, and excessive foot pronation. This condition does not produce an effusion. Anterior and posterior ligament tears would be highly unlikely given that there was no history of an acutely traumatic event and because ligaments are fairly vascular structures, an effusion would be expected with any cruciate ligament tear. Meniscal tears in a young patient are generally associated with a traumatic twisting event and while not as vascular as a ligament, meniscal tears generally lead to the gradual development of an effusion. Older patients may experience a degenerative tear with minimal trauma, but that is unlikely in our 17-year-old patient. Osteoarthritis is generally associated with gradual wear and tear after many years or the process can be accelerated after a significant acute trauma to a joint. Neither scenario is applicable to this patient.

A 24-year-old HIV-positive man comes to the emergency department complaining of severe left-sided chest discomfort, which radiates through to the left trapezius region. On coming into the room, you note that he is sitting up and hunched forward. On physical examination, the patient's blood pressure is 135/78, with a pulse of 85 bpm, and a pericardial friction rub is noted. Laboratory findings demonstrate elevated serum creatine kinase levels and normal serial troponin levels. His EKG demonstrates peaked T waves. His CXR demonstrates a "water bottle" cardiac silhouette. Which of the following are serious consequences of acute pericarditis, which require careful monitoring? A Pericardial effusion B Aortic dissection C Myxedema D Chylopericardium E Acute myocardial infarction

pericardial effusion A Choice A, pericardial effusion, is a serious consequence of acute pericarditis, which requires careful monitoring to ensure that progression of the pericardial effusion does not lead to cardiac tamponade, which can be fatal if not treated promptly. Choice B is unlikely in a patient with acute pericarditis. Choices C, D, and E are noninfectious causes, not consequences of pericarditis.

A 24-year-old HIV-positive man comes to the emergency department complaining of severe left-sided chest discomfort, which radiates through to the left trapezius region. On coming into the room, you note that he is sitting up and hunched forward. Prior to examining him, you have reviewed his chart. Laboratory findings demonstrate troponins x 3, which are negative for myocardial ischemia. His EKG demonstrates diffuse ST segment elevations throughout. Which of the following physical exam findings would be most likely in this patient? A Roth spots B Splenic enlargement C Janeway lesions D Pericardial friction rub E Splinter hemorrhages

pericardial friction rub D Choice D is the most likely finding, as this patient is exhibiting signs, symptoms, and EKG findings pathognomonic for acute pericarditis, which is likely infectious in the setting of a patient with HIV. A pericardial friction rub is heard best with the patient in a seated position, during expiration, and is frequently found in patients with pericarditis. Choice A B, C, and E are physical exam findings seen in acute bacterial endocarditis

An 89-year-old female was found to have had an episode of syncope while at home. There was no history prior to the event, and the patient denies any chest pain prior to the episode occurring. On exam, the patient is alert, awake, and oriented, and is only complaining of mild shortness of breath. Her ECG is as shown. Which of the following is the best choice for treating this patient? EKG shows 3rd degree AV block. A No treatment B Permanent pacemaker C Digitalis therapy D Ablation therapy E Beta blockers

permanent pacemaker This patient has a third-degree AV block that is symptomatic, with syncope and mild shortness of breath. The treatment for this patient is pacemaker insertion. Immediate insertion depends on the vital signs of the patient. If the patient's vital signs are stable, a transdermal pacer can be used until a scheduled pacer insertion within 24 hours. If the vital signs are not stable, immediate intervention with either central venous pacing wires or an immediate pacer insertion is warranted.

A 53-year-old man presents to the emergency department because of fever, headache, and confusion. On physical examination, you note an obtunded man who appears acutely ill with temperature of 104°F, blood pressure of 128/76 mm Hg, pulse of 98, and respiratory rate of 20. The patient has stomatitis, nuchal rigidity, and a positive Kernig sign. CSF examination shows increased opening pressure, 80 WBC/mL (normal < 10/mL), mildly elevated protein, and normal glucose. Which of the following tests would confirm the most likely causative organism? A CT of the head B polymerase chain reaction test for herpes simplex virus C blood culture for herpes simplex virus D serum IgG for herpes simplex virus E MRI of the head

polymerase chain reaction test for herpes simplex virus B The patient's presentation is consistent with viral meningitis with potential encephalitis. The presence of active stomatitis indicates herpes simplex virus as the most likely causative organism. A CT of the head could be considered prior to performing a lumbar puncture and may show temporal lobe abnormalities that support a diagnosis of herpes virus encephalitis, but like an MRI will not identify the causative organism and has limited sensitivity. Of the three herpes tests described, the PCR technique is the most likely to identify the herpes simplex virus as the causative organism in the CSF due to its high sensitivity and specificity. Serum IgG indicates prior infection from herpes simplex virus but does not confirm the causative organism of the patient's encephalitis. Viral blood cultures for herpes simplex would likely show no growth even in the presence of herpes simplex virus encephalitis

In addition to insulin and fluid replacement with 0.9% saline, which electrolyte is commonly infused in the type 2 diabetic patient who arrives in the emergency department in a hyperglycemic, hyperosmolar, nonketotic state? A bicarbonate B potassium C calcium D magnesium E sulfate

potassium Insulin not only causes cellular uptake of glucose but also of potassium. Hypokalemia may develop when insulin is infused to correct either a hyperglycemic hyperosmolar state or a diabetic ketoacidosis. Hence, in order to avoid hypokalemia, potassium chloride can be added to a saline solution, as long as the serum potassium is not elevated. (Masharani, 2008, pp. 1065-1067) Masharani U. Diabetes mellitus & hypoglycemia.

A 1-year-old boy is brought to the emergency department by his parents, who state that the child refuses to walk or crawl and begins crying when they stand him. Swelling to his right knee is noted; it is also warm to the touch and pain response is noted. His parents state that it seemed to start a couple of days ago and has gotten worse. They don't recall a trauma, but state that he seems to bruise easily. The child's mother states that she also bruises easily. Vitals are as follows: Temp: 38.0°C, HR: 70, RR: 15. What laboratory finding would you expect? A Prolonged aPTT (activated partial thromboplastin time) B Prolonged bleeding time C Prolonged PT (prothrombin time) D Prolonged thrombin clotting time E Thrombocytopenia

prolonged aPTT This patient has hemophilia A. Patients with severe hemophilia A have a prolonged aPTT; all of the other tests should be within the normal range.

A 45-year-old man presents with hematemesis. He has had 2 episodes of vomiting 'coffee-ground'-appearing material; the vomiting began 45 minutes prior to presentation. Additionally, he reports passing black, sticky stools for the past 3 or 4 days. Past medical history is positive for occasional headaches; they have been coming more frequenly lately. Social history reveals alcohol use (1 case of beer each weekend) and tobacco (1 pack per day). Medications include ibuprofen as needed for headaches; he has been taking 800 mg 3 times a day for the past week. You place a nasogastric tube and find bright red blood that fails to clear with saline irrigation. Hemoglobin is 8.9 g/dL. Evaluation of his blood pressure and pulse reveals orthostatic changes that resolve with an intravenous fluid bolus of 500 cc of Lactated Ringer's solution. What should you do next? Answer Choices 1 Transfuse 2 units of packed red blood cells and recheck CBC in 8 hours 2 Check serum for Helicobacter pylori antibody 3 Order a double contrast barium swallow to evaluate for a gastric ulcer 4 Refer for emergency upper endoscopy 5 Discontinue his ibuprofen use and encourage cessation of alcohol and tobacco

refer for emergency endoscopy He should be referred for an emergency upper endoscopy. This patient is most likely bleeding from a gastric ulcer. His recent NSAID use, as well as his alcohol and tobacco habits, make him at risk for peptic ulcer disease. His symptoms of melena and hematemesis, along with his anemia, make the diagnosis quite straightforward. It appears that this patient is still actively bleeding based on the results of the nasogastric tube irrigation; therefore, the priority should be getting the ulcer to stop bleeding. Upper endoscopy should be performed so that the bleeding site can be identified and treated with electrocautery, coagulation, or injection of epinephrine or a sclerosing agent. If the bleeding cannot be stopped with endoscopic interventions, angiographic embolization should also be tried. If these interventions do not succeed, the patient has rapid deterioration, or if he requires more than 6 units of blood in a 24-hour period, then emergency surgery may be indicated. The other choices are not the best options for immediate management. This individual cannot be followed simply with transfusions and serial CBC's because he appears to still be actively bleeding. Helicobacter pylori infection may very well be playing a part in the etiology of this man's ulcer, but evaluation for H. pylori can be done with a biopsy at the time of his endoscopy; it will not help in his immediate management. A barium esophagram will not identify actively bleeding ulcers and cannot treat active bleeding. While NSAID, alcohol, and tobacco use may have precipitated this man's GI bleed, counseling about his use of these substances will not sufficiently treat his immediate bleed.

A 5-year-old male is being evaluated for an acute injury to the right ankle. On the x-ray of the ankle there is a distal tibia fracture that involves the separation of the epiphysis, as well as a small non-displaced chip fracture of the metaphysis of the tibia. Based on these findings, what type of Salter-Harris fracture does this child have? A I B II C III D IV E V

salter harris 2 The growth plate is the most fragile part of the bone prior to bone maturation and thus is usually the first structure disrupted when force is applied. Statistically, Type II fractures are most common - those that involve both the growth plate and a chip fracture of the metaphysis.

A 78-year-old female trips and falls on an outstretched hand and now presents with right wrist pain. On examination, the patient complains of pain on palpation to the right radial side of the wrist near the anatomical snuffbox. Based on this clinical presentation, what is the most likely fracture that this patient has? A Capitate B Lunate C Scaphoid D Trapezium E Trapezoid

scaphoid The scaphoid or navicular bone of the wrist, is the most commonly fractured carpal bone. This injury occurs most commonly in young men. The lunate is the second most commonly fractured carpal bone and the most commonly dislocated carpal bone. The capitate is the largest of the carpal bones and it is not known as a common isolated fracture site. Fractures of the trapezium and trapezoid are relatively uncommon as well

A 22-year-old woman, with no previous medical problems, suddenly cried out, fell to the ground, extended her legs, flexed her arms, and jerked her extremities for 30 seconds. There was associated tongue biting and urinary incontinence. She awoke slowly over a 10-minute period and recalled nothing about the episode. She remained lethargic for several hours but the rest of her neurologic examination was normal. What is the most likely etiology for this episode? A epilepsy B hyperventilation C cardiac arrhythmia D seizure E stroke

seizure D This event represents a well-demarcated episode affecting some combination of consciousness, motor, and/or sensory function consequent to abnormal electrical discharges in the brain. This is consistent with the definition of a seizure. Epilepsy refers to multiple, recurrent seizures. This history is not consistent with hyperventilation, stroke, or cardiac arrhythmia, which would typically include chest pain, shortness of breath, dyspnea on exertion, or focal neurological deficits.

A 24-year-old intoxicated male presents to the emergency department after being in a fight. He was punched in the nose, and now has mild deformity of the nose and some epistaxis. An x-ray reveals a fractured nasal bone. During his physical exam, what must you look for in order to prevent permanent destruction of his nasal septum? A Orbital fracture B Posterior epistaxis C Septal hematoma D Facial fracture

septal hematoma A septal hematoma can cause ischemic necrosis of the nasal septal cartilage if not identified and drained. A deviated septum can be expected with a nasal bone fracture, and must be addressed by the otolaryngologist. Excessive epistaxis that does not resolve with direct pressure and anterior packing may indicate a posterior bleed.

A 32 year-old male presents to the emergency department in Acute Renal Insufficiency (AKI). Which of the following conditions would be most likely observed in intrinsic AKI? A Septic shock B Congestive heart failure C Benign prostatic hypertrophy D NSAID overdose E Chronic liver failure

septic shock The most common causes of intrinsic AKI are sepsis, ischemia, and nephrotoxins, both endogenous and exogenous. Prerenal acute kidney injury can be caused from hypovolemia, decreased cardiac output, decreased circulation of blood volume (CHF, liver failure), and impaired renal autoregulation (NSAIDs, ACE-I/ARB, cyclosporine)---(E), (D), and (B). Postrenal causes include bladder outlet obstruction including bladder stones and BPH (C).

What is the most common ECG abnormality in patients with a pulmonary embolism (PE)? A Atrial fibrillation B Sinus tachycardia C Ventricular ectopy D Sinus bradycardia

sinus tachycardia B In most cases, sinus tachycardia is the only abnormality in patients with a PE. You may also find some ECGs that will have non-specific ST-T wave changes. Sinus bradycardia and AV blocks are not common findings that are associated with PE.

Which of the following is the initial treatment step in an adolescent who presents to the emergency department with status epilepticus? A IV glucose B stabilize airway C arterial blood gas D IV diazepam therapy

stabilize airway Status epilepticus is a medical emergency and is defined as seizure activity that lasts a minimum of 30 minutes. This results in hypoxia, acidosis, cerebral edema, and structural damage. In addition, fever, respiratory depression, hypotension, and death may occur. There are both convulsive and nonconvulsive types of status epilepticus. Because of its emergency status and potential complications, the clinician needs to initiate the ABCs (airway, breathing, circulation). Therefore, the first line of treatment is to establish and maintain an airway, oxygen is next, and then circulation, which encompasses pulse, blood pressure, and IV access. Once the IV is established, the orders should be for administering glucose-containing fluids and IV drug therapy with diazepam, lorazepam, or midazolam as well as administer phenytoin and phenobarbital. Arterial blood gases should be ordered and any abnormalities should be corrected appropriately. Finally, the clinician should determine the underlying cause: trauma, structural disorder, infection, lactic acidosis, toxins, and uremia. Maintenance drug therapy is necessary until the underlying cause is determined and rectified.

A 76-year-old male presents to the hospital with a hot, swollen, and painful right knee that appears to have an effusion. After your examination, you decide to perform an arthrocentesis. The fluid aspirate is turbid, cloudy, and most definitely looks like it is infectious material. Based on this clinical scenario, what would be most likely bacterial organism present in an elderly septic arthritis? A Hemophilus influenzae B Neisseria gonorrhoeae C Pseudomonas aeruginosa D Staphylococcus aureus

staph aureus D Septic arthritis is an infection within a joint space. The infection can be caused by multiple pathogens including bacteria, viruses, fungi and mycobacteria, but in older adults the most common organism implicated is Staphylococcus aureus. These bacteria can get into the joint space through multiple mechanisms including direct inoculation after a penetrating injury, hematogenous spread from an infection elsewhere in the body, or from extension from a contiguous bone infection. All of the bacteria mentioned as possible answers are possible causes of septic arthritis, but there prevalence varies within different patient populations. Hemophilus influenza and Streptococcus pneumonia are commonly seen in children less than five years of age that develop septic arthritis. Neisseria gonorrhoeae is most common in sexually active adolescents and young adults. Pseudomonas aeruginosa is a common cause of septic arthritis in IV drug abusers

Which rotator cuff muscle is most commonly injured? A Infraspinatus B Subscapularis C Supraspinatus D Teres Major E Teres Minor

supraspinatus C The supraspinatus is involved in abduction and external rotation of the shoulder joint. It is often injured in repetitive overhead activities such as swimming and throwing sports. While all muscles of the rotator cuff can be injured, the supraspinatus has been shown to sustain the most frequent injuries. The infraspinatus and teres minor are also involved in abduction and external rotation, while the subscapularis and teres major assist in internal rotation. The teres major is not considered a rotator cuff muscle

A 54-year-old man presents to the emergency department with crampy abdominal pain, nausea, and vomiting. The patient has not passed gas or had a bowel movement for at least 10 hours. On examination, the abdomen is distended and there are high-pitched bowel sounds with rushes. A plain radiograph of the abdomen reveals cecal distension to 12 cm. What is the most appropriate definitive management for this patient? A Intravenous fluids B Nasogastric suction C Observation D Surgical exploration

surgical exploration Massive distention of the cecum, as detected on plain radiograph, is typically seen in "closed loop" obstructions where the ileocecal valve is competent. When distention approaches 12 cm, there is an increased risk of perforation and/or gangrene. Expedient surgical intervention is indicated. Although observation with intravenous fluids and nasogastric decompression are important adjuncts to management, surgical exploration is the only way to rapidly address this emergent situation.

A 44-year-old female presents to the emergency department with a right-sided headache. She states the headache is located on the right temple region, is non-radiating, and does not cause photophobia. She is otherwise healthy and has no reported medical problems. She only takes acetaminophen for the pain, which has minimal relief. On physical exam she is alert, awake, and oriented. Her vitals are T 98.8, P 78, R 18, and BP 128/76. Her head is normocephalic, atraumatic, and pupils are equal and reactive. She has tenderness to the right temporal area of the temporal artery, and there is no noted swelling, redness, or abnormalities noted. There is no tenderness to the cervical muscles. She exhibits a non-focal neurological exam. Based on the description, what is the most likely diagnosis of this patient? A Migraine headache B Tension headache C Cluster headache D Temporal arteritis E Takayasu arteritis

temporal arteritis D This patient is exhibiting a case of temporal arteritis (D). She has the signs and symptoms that are classic in nature and do not fit into the realm of the other headaches (A, B, and C), all of which usually have different characteristics on history and physical exam. Takayasu arteritis (E) will typically not present in the temporal artery.

A 14-year-old boy presents to the emergency department with acute scrotal pain and vomiting for the past 2 hours. His left testicle is in extreme pain and he states the pain started while playing basketball in gym class. Which diagnosis is highest on your differential in regards to his clinical presentation? A Epididymitis B Hydrocele C Testicular torsion D Varicocele E Intrascrotal tumor

testicular torsion C Testicular torsion (C) is most common between ages 12-18 with the classic presentation of abrupt and severe onset of pain with nausea/vomiting. The most common cause of painless scrotal swelling in children is a hydrocele (B). Epididymitis (A) usually has an insidious onset, commonly presenting with urinary frequency, dysuria, or fever. Varicoceles (D) and intrascrotal tumors (E) usually present as painless scrotal edema.

A patient presents to the emergency department with signs and symptoms of acute pancreatitis. He does not drink alcohol and has no history of gallstones. He has a positive history of increased lipids. Very high levels of which of the following lipids is associated with an increased risk of pancreatitis? A High density lipoprotein B Low density lipoprotein C Apolipoprotein C D Total cholesterol E Total triglycerides

total triglycerides The correct choice is E, total triglycerides. Very high levels of triglycerides increase the risk of acute pancreatitis, probably from local release of free fatty acids and lysolecithin from lipoprotein substrates in the pancreatic bloodstream. If the albumin binding capacity is exhausted, these fatty acids can initiate a chemical pancreatitis and destroy parenchymal cells in the area.

A 33-year-old IV drug user presents to the emergency department with pleuritic chest pain, cough, chills, diaphoresis, anorexia, and malaise. On physical exam, her temperature is 40°C, BP 98/55, P 115 bpm, and RR 22. No murmur could be appreciated. Two separate blood cultures are positive for S.aureus. An EKG, CXR, and transesophageal echocardiogram are ordered. Which of the following lesions is most likely to be seen on TEE in this patient? A Aortic valve vegetation B Tricuspid valve vegetation C Mitral valve vegetation D Left ventricular hypertrophy E Ventricular septal defect

tricuspid valve vegetation In almost 50% of cases involving IV drug users, the only site of infection is the tricuspid valve, and most lesions are right-sided, so choice B is the most appropriate answer. Left ventricular hypertrophy, choice D, is seen in patients with a history of hypertension. Choice E, ventricular septal defect, is frequently associated with a holosystolic murmur.

A fracture involving the medial epicondyle will most likely cause damage to which nerve? A Axillary B Median C Peroneal D Radial E Ulnar

ulnar Because the ulnar nerve passes through the cubital tunnel, which is a groove on the posterior aspect of the medial epicondyle, any fractures involving the medial epicondyle can also cause damage to the ulnar nerve. The median nerve is most susceptible to injury at the carpal tunnel. Branches of the radial nerve can become entrapped on the lateral side of the elbow and the associated symptoms are often confused with lateral epicondylitis. Radial nerve injuries are more commonly associated with humeral shaft fractures. The axillary nerve is significantly proximal to the medial epicondyle and the peroneal nerve is in the leg

A 59-year-old woman presents to the emergency department with an acute upper gastrointestinal hemorrhage. Her medical history is pertinent for peptic ulcer disease for the past 5 years and hypertension. A nasogastric tube is inserted and bright red blood is seen. Her vital signs are BP: 110/70 mm Hg, P: 94/min, R: 14/min, oxygen saturation: 97%, T: 99°F. Which of the following diagnostic studies would be the most appropriate next step to determine the site of bleeding? A Abdominal and pelvic computed tomography B Abdominal ultrasound C Upper gastrointestinal series with barium D Bleeding scan E Upper endoscopy

upper endoscopy Patients who present with upper gastrointestinal bleeding and shock requiring multiple transfusions in 24 hours are at high risk for mortality from gastrointestinal bleeding. The upper gastrointestinal bleeding in this patient warrants further investigation with upper gastrointestinal endoscopy to both determine the site of bleeding and provide potential therapy by endoscopic electrocautery or injection.

What is the most common pathogen that causes an acute bronchitis? A Viral B Bacterial C Fungal D Unknown

viral A Respiratory viruses are the most common cause of acute bronchitis. In clinical medicine, it is rare to obtain cultures for patients who present with bronchitis symptoms.

A 70-year-old woman who was found barely responsive at home by her daughter is brought to the emergency department. Evaluation reveals that she is in a hyperglycemic hyperosmolar state with a severe fluid deficit. Treatment is initiated with vigorous saline rehydration and a continuous infusion of insulin. At what point should her glucose be added to her treatment? A when her condition becomes stable B when her urine output reaches 50 mL/hour C when her blood glucose reaches 250 mg/dL D if she develops hypokalemia E if she begins to spill ketones in her urine

when her blood glucose reaches 250 mg/dL In hyperglycemic hyperosmolar states, the serum glucose rapidly corrects with fluid administration alone. However, with vigorous rehydration, glucose may fall precipitously and lead to severe hypoglycemia. To avoid this, glucose should be added to water, half-normal, or normal saline as soon as the patient's blood glucose is less than or equal to 250mg/dL. She should continue to receive insulin IV until she is stabilized (A) when it can be switched to subcutaneous administration. The goal of fluid therapy in this patient is restoring her urine output to 50 mL per hour (B) or more. Because insulin drives potassium into the cells and can cause hypokalemia (D), potassium chloride should be given unless the patient has chronic kidney disease or oliguria. Persons in a hyperglycemic hyperosmolar state typically do not spill ketones (E) the way persons with diabetic ketoacidosis do.

A 66-year-old male with a history of hypertension, diabetes mellitus, and hypercholesterolemia presents by emergency medical services (EMS) to the emergency department complaining of severe chest pain with radiation into his back. The patient states that he was feeling well in the morning, but while performing some light activity he felt a "ripping" sensation in his back, which he initially thought was a pulled muscle. The pain continued and the patient started to have chest pain, shortness of breath, and lightheadedness. On initial examination the patient is still in pain, pale, diaphoretic, and has a blood pressure of 85/40. His chest is clear to auscultation, and he has a 3/6 diastolic murmur best appreciated at the base of the heart. Given this clinical scenario, what would be the expected finding on chest x-ray? A Normal B Pleural effusion C Decreased lung volume D Widened mediastinum E Cardiomegaly

widened mediastinum

A 44-year-old female presents to the emergency department with a right sided headache. She states the headache is located on the right temple region, is non-radiating, and does not cause photophobia. She is otherwise healthy and has no reported medical problems and has only taken acetaminophen for the pain, which has minimal relief. On physical exam she is alert, awake, and oriented. Her vitals are T 98.8, P 78, R 18, and BP 128/76. Her head is normocephalic, atraumatic, and pupils are equal and reactive. She has tenderness to the right temporal area of the temporal artery, and there is no noted swelling, redness, or abnormalities noted. There is no tenderness to the cervical muscles. She exhibits a non-focal neurological exam. A biopsy of the temporal artery reveals inflammatory cells of the artery wall. Based on the history and biopsy results, what is the best initial treatment for this patient? A Cyclosporine B Azathioprine C Methotrexate D Ibuprofen E Prednisone

Prednisone E This patient is exhibiting a case of temporal arteritis. She has the signs and symptoms that are classic in nature and do not fit into the realm of the other headaches, all of which usually have different characteristics on history and physical exam. In this instance, the use of oral steroids will be the best choice in management of the condition. Anti-inflammatories (C, D) and immunosuppressant agents (A, B) will not be as effective as oral prednisone (E), and the long-term outcomes are improved with steroids.

A 42-year-old woman with a history of migraine cephalgia and Raynaud's phenomenon comes to the emergency department with complaints of severe chest discomfort that occurs at rest every morning (at approximately 10 AM). An EKG performed during an episode of chest discomfort demonstrates transient ST segment elevation, which is relieved with sublingual nitroglycerin. There is no troponin elevation. Cardiac catheterization is performed, and reveals coronary artery spasm, which corresponds with ST segment elevation, and no significant coronary artery stenosis. Which of the following choices is the most likely diagnosis? A Pericarditis B Acute myocardial infarction C Costochondritis D Prinzmetal angina

Prinzmetal angina D Prinzmetal angina, or variant angina pectoris, is defined as coronary artery spasm associated with ST-segment elevation, and usually occurs at rest and at the same time of the day. Patients with a history of migraine cephalgia and Raynaud's phenomenon demonstrate Prinzmetal angina more frequently than the rest of the patient population. This can occur in patients with normal coronary arteries and with coronary artery stenosis. Choice A, pericarditis, would present with chest discomfort that is worse while supine and improves with sitting up, as well as a pericardial friction rub. Choice B, acute myocardial infarction, would present with troponin elevation, and is unlikely in the setting of a patient with normal coronary arteries on cardiac catheterization. Choices C and E would not be relieved with sublingual nitroglycerin or demonstrate transient ST-segment elevation.

A 65-year-old male presents with pain and swelling to his right knee without any history of injury. He has had this type of pain and swelling before, and does recall that he had fluid drained out of the knee several years ago. On examination the patient has a swollen, tender knee with a palpable effusion. There is decreased range of motion to the joint secondary to the effusion. An arthocentesis is performed, and the analysis of the fluid reveals calcium pyrophosphonate crystals. Based on these findings, what is the most likely diagnosis? A Septic arthritis B Acute synovitis C Hemarthoma D Pseudogout

Psuedogout D Pseudogout is also known as Calcium Pyrophosphate Deposition Disease (CPDD) and most commonly affects patients over the age of 65. The knee joint is most commonly affected with the wrist joint second most common. An examination of synovial fluid aspirated from the affect joints, will reveal calcium pyrophosphate crystal. Ankle joints are rarely affected by pseudogout but are fairly common locations for gout caused by urate crystal deposition. The first metatarsal phalangeal joint is the classic location for gout (sometimes referred to as podagra), but it is not a typical pseudogout location. The glenohumeral joint of the shoulder is affected by pseudogout much more than gout. Distal interphalangeal joints of the hand are rare locations for either gout or pseudogout, but are classic locations to see signs and symptoms of osteoarthritis. When pseudogout does affect the hands it is generally seen in the metacarpophalangeal joints.

A 36 year-old woman with no significant past medical history presents with gradual onset of dyspnea and fatigue leading to an episode of "fainting" this morning. Physical exam reveals increased jugular venous pressure, weak carotid pulses, clear lungs, and a loud S2. What is the most likely diagnosis? A Aortic stenosis B Cardiac tamponade C Mitral Regurgitation D Pulmonary fibrosis E Pulmonary hypertension

Pulm HTN The patients symptoms are due to decreased cardiac output resulting from decreased preload associated with pulmonary hypertension (E). Aortic stenosis (A) presents more commonly in geriatric patients who present with a murmur. Cardiac tamponade (B) can decrease cardiac output, but would lead to decreased heart sounds. Mitral regurgitation (C) would cause pulmonary edema and rales in conjunction with increased jugular venous pressure. Pulmonary fibrosis (E) is unlikely in this patient with normal lung sounds.

An 18 month old female is brought to the pediatricians office with a history of cough, fever of 102, and decreased fluid intake. Her immunizations are not up to date as the family just moved to the United States from out of the country. On physical exam she is drooling and sitting up in a "tripod position" with mild stridor. What is the most appropriate treatment indicated for this condition? A Humidified air B Albuterol nebulizer C Budesonide nebulizer D Recemic epinephrine nebulizer E Ipratropium nebulizer

Racemic epinephrine nebulizer The clinical presentation suggests epiglotitis. This is an emergent airway condition. The anesthesiologist , or the pediatric otolaryngologist must be called to stand by to intubate or insert a tracheostomy if the patients airway closes. Racemic epinephrine via nebulizer relieves much of the edema to the upper airway in a patient with epiglotitis. It is a stabilizing measure until definitive care can be arranged. Oxygen and antibiotics should administered emergently also. No x-rays are indicated when the presentation is classic. Albuterol is a beta-agonist used for treatment of asthma. Budesonide,a steroid and ipratropium, an anticholinergic agent are most often used in combination with albuterol for treatment emphysema and asthma.

What nerve is most commonly injured in a mid- or distal humeral shaft fracture? A Axillary B Median C Peroneal D Radial E Ulnar

Radial Because of the radial nerves proximity to the humerus, mid and distal shaft fractures with significant displacement can cause a radial nerve injury. Median and ulnar injuries are more commonly associated with forearm injuries. Axillary nerve injuries are most common in anterior shoulder dislocations and peroneal nerve damage occurs as a result of lower leg insult

A 23-year-old female has been seen in your Emergency Department after being beaten by her husband. As you counsel her before she leaves, which of the following will you recommend? A Avoiding the behavior that brought on the attack B Couples counselling C Leaving the relationship D Prosecuting her husband E Referral to a local women's shelter

Referral to a women's shelter E The appropriate course of action when working with a person who has suffered intimate partner violence is to validate his or her experience, document clearly and non-judgmentally, and assess immediate safety. Referrals to appropriate resources (E) should be made, but decisions regarding the relationship (B, C) and any legal action (D) should be left to the patient rather than continuing a pattern of controlling behavior. Suggesting she avoid behaviors that provoke her attacker (A) puts the blame on the victim.

A 77-year-old male is admitted to the ICU with community acquired pneumonia and sepsis. There is a concern for the possible development of stress gastritis. Which of the following is an important risk factor in the development of stress gastritis? A Platelets < 150,000 per microliter B INR < 1.5 C Patient remaining NPO for > 24 hours D Hematocrit < 35% E Respiratory failure requiring mechanical ventilation > 48 hours

Respiratory failure requiring mechanical ventilation > 48 hours Two of the most important risk factors in the development of stress gastritis are coagulopathy (platelets < 50,000/mcl or INR > 1.5) and respiratory failure, requiring mechanical ventilation for > 48 hours. Hematocrit and amount of time that a patient receives nothing by mouth are not important risk factors.

An 8-year-old girl is rushed to the emergency department by her parents because she has become delirious. The child was diagnosed with influenza three days prior. Her parents say that she had begun vomiting yesterday, almost nonstop, and has not been able to hold down fluids. They also note that she has been breathing rapidly. Your exam reveals a tachypneic, disoriented female with hyperreflexia, a positive babinski reflex, and liver enlargement. CSF analysis reveals a normal protein and cell count. What is the most likely diagnosis? A Bacterial meningitis B Guillain Barre syndrome C Measles encephalitis D Reye's syndrome E Viral meningitis

Reye's syndrome Although rare, Reye's syndrome is associated with viral infections, salicylate use during illness, and metabolic disorders. Illness is associated with liver fat deposition and degeneration, intractable vomiting, and mental status changes, which may progress to seizures, delirium, and coma. Cerebral edema contributes to these changes and other neurologic findings. Meningeal signs are more consistent with meningitis. Measles encephalitis typically presents days to weeks after the pathognomic measles exanthem and clinical findings. Guillain Barre has been associated with influenza infection, and signs and symptoms would include evolving weakness with ascending paralysis and extremity dysesthesias.

An 18 month infant with congenital heart disease is diagnosed with acute bronchiolitis secondary to respiratory syncytial virus. Which of the following therapies should be initiated? A Albuterol B Amoxicillin C Azithromycin D Prednisone E Ribavirin

Ribavirin High-risk infants (i.e., congenital heart disease) who develop RSV are eligible for treatment with ribavirin. Antibiotics (B, C) are indicated if secondary bacterial infections develop. Albuterol (A) and prednisone (D) haven't been shown to improve RSV-related bronchiolitis.

A 39-year-old male patient presents with low back pain with radiation to the right leg. On examination you place the right hip in a flexed position, and as you palpate between the iscial tuberosity and the greater tronchanter of the femur the patient complains of radiation of pain down his right leg. Based on this history and exam, which nerve was affected by this part of the examination maneuver? A Femoral B Peroneal C Saphenous D Sciatic E Sural

Sciatic D The sciatic nerve does lie midway between the ischial tuberosity and greater trochanter and it can be palpated when the patient is in a hip flexed position. The gluteus maximus obscures the nerve from being effectively palpated when the leg is in an extended position. Tenderness of the sciatic nerve can be caused by a lumbar disk herniation, direct trauma, or spasm of the nearby pyriformis muscle. The femoral nerve is a deep structure that lies lateral to the femoral artery and is not considered to be palpable. The femoral nerve is responsible for the L1-3 dermatomes and for supplying motor function to the iliopsoas muscle. The peroneal nerve originates from the sciatic nerve and splits into the superficial and deep peroneal nerves, which are responsible for much of the sensory and motor nerve function in the lower leg. The saphenous nerve originates from the femoral nerve in the femoral triangle and runs down the medial aspect of the leg. The sural nerve has medial and lateral components that are found in the lower leg. The medial cutaneous sural nerve arises from the tibial nerve just below the knee and eventually connects with peroneal nerve to form the sural nerve. On the lateral side of the lower leg, the sural nerve arises from the common peroneal nerve just above the knee and eventual connects with the previously discussed medial branch to form the sural nerve

A 16-year-old male soccer player is complaining of pain to the right foot that has been getting progressively worse for the last 2 months. He states it hurts the most when he has all of his weight on his right foot as he plants to kick the ball. Most of the pain appears to be on weight bearing. You are concerned that this patient may be developing a stress fracture. Based on the patient's history and patient presentations, which bone is the most affected by stress fractures in the foot? A Calcaneus B Fifth metatarsal C First metatarsal D Second metatarsal E Talus

Second Metatarsal Any bone that is exposed to repetitive stress can have a stress fracture, but the long and thin metatarsal bones of the foot are the most commonly affected bones. Of the metatarsals, the second metatarsal has the highest number of stress fractures. These weight bearing bones can be particularly vulnerable to stress fracture if the patient is involved in long distance running, especially if he/she is wearing improper footwear for that activity or footwear that has lost most of its shock absorbing abilities. Some young female athletes may be training so hard that they become amenorrheic which can contribute to osteopenia resulting in weaker bones. Older patients with osteoporosis will also have a higher risk of stress fracture. Initially stress fractures of the metatarsals may present with a small area of localized pain and the dorsal forefoot may demonstrate a fairly diffuse area of swelling. If the stress fracture is not treated early, some patients will experience an audible pop or crack as the incomplete stress fractures progresses to a complete break. All types of fractures occur more easily in long thin bones like the metatarsals, than thicker bones like the calcaneus and talus

A 68-year-old woman with a history of hypertension and diabetes mellitus type 2 comes to the emergency department with her son, who noticed that while decorating for Christmas she seemed more dyspneic than normal, and had to sit down frequently. In addition, he noticed that she was pale and diaphoretic, and insisted on driving her to the emergency department. On questioning, she denies chest pain, but admits to being more fatigued than usual, with frequent jaw discomfort during activity. Activities such as vacuuming her house cause dyspnea, and she now has to stop several times while carrying laundry up from the basement. On physical examination, the patient's blood pressure is 90/50, pulse 99 bpm, respirations 22, and she is afebrile. Auscultation of the chest demonstrates a new systolic murmur. An EKG demonstrates normal sinus rhythm with nonspecific ST and T wave changes. Which of the following would be the most appropriate next step in the management of this patient? A Transesophageal echocardiogram B Nuclear stress test C Cardiac catheterization D Serial serum troponin levels E CXR

Serial serum troponin levels Choice D, checking serial serum troponin levels, is the most appropriate next step in the management of this patient. Women and diabetics may present with atypical symptoms with acute non-ST-segment myocardial infarction, including dyspnea, jaw discomfort, and epigastric discomfort. Frequently, women present much later than men with these symptoms. Therefore, a high level of suspicion should be maintained when women present with symptoms of dyspnea, even in the setting of nonspecific EKG changes, and drawing serum troponin levels before any other testing is recommended. Once non-ST-segment myocardial infarction has been ruled out, choices E and B, and also transthoracic echocardiogram, would likely be evaluated. Transesophageal echocardiogram may be required if better visualization of the heart valves is required, but not as the next step. Choice C, cardiac catheterization, would likely occur if an abnormal stress test demonstrating symptoms of myocardial ischemia is found.

An 80-year-old male nursing home patient is brought to the emergency department with abdominal distension. A plain film of the abdomen is pictured below. Which of the following is the most likely diagnosis? Picture of a "coffee bean" looking abdominal x-ray. A Small bowel obstruction B Cecal volvulus C Sigmoid volvulus D Toxic megacolon

Sigmoid Volvulus A volvulus is an obstruction of the colon due to a loop of bowel that has rotated more than 180 degrees on its axis with the mesentery. The most common site for a volvulus is the sigmoid colon (65%). A sigmoid volvulus is associated with abdominal pain and distension. Plain films of the abdomen would show a characteristic "bent inner tube" appearance. Sigmoidoscopy can be used to decompress the bowel by gently releasing the area of obstruction. Following decompression, a rectal tube is inserted to act as a stent to prevent the bowel from twisting upon itself again.

A 42-year-old healthy male presents to the emergency department with the complaint of a progressively worsening sore throat, and difficulty swallowing over the past 48 hours. He also complains of a subjective fever, but denies any headaches, nausea, or vomiting. On exam, the patient is afebrile and in mild distress, with a presentation of leaning forward on the exam table. His TM examination is normal, there is no rhinorrhea, and the oropharynx is patent without signs of stridor. His lungs are clear, and he has a regular rhythm on cardiac exam. What diagnostic test is indicated for a definitive diagnosis? A Chest x-ray B Complete blood count (CBC) C Nasal bacterial culture D Soft tissue neck x-ray E CT scan of the neck

Soft tissue neck Xray D This patient has a presentation that is consistent with acute epiglottitis. While ensuring that the airway is patent and the patient can maintain the airway, the first step in determining the diagnosis is a soft tissue neck x-ray, to determine inflammation to the epiglottis. While rare, epiglottitis can be from a bacterial infection, and can be quite serious and sometimes fatal.

What type of fracture is not related to an acute bony trauma? A Greenstick B Stress C Oblique D Comminuted E Spiral

Stress B A stress or fatigue fracture is caused by small, repetitive forces that usually involve the metatarsal shafts, the distal tibia, and the femoral neck (though many other bones may be affected). These fractures may not be seen on initial radiographs. A greenstick fracture is an incomplete traumatic fracture with angular deformity seen in children. An oblique fracture is a traumatic fracture with an angulated fracture line. A comminuted fracture is a traumatic fracture in which there are more than two fracture segments. A spiral fracture is a traumatic fracture that has a multiplanar and complex fracture line usually caused by an excessive rotational force on a bone.

A 1-year-old boy is brought to the emergency department by his parents, who state that the child refuses to walk or crawl and begins crying when they stand him. He seems calm while lying on the examination table. Vitals are as follows: Temp: 38°C, HR: 70, RR: 15. Bruising is noted in several places. His parents deny trauma, but have noticed that he bruises easily. What other physical finding would you expect? A Conjunctival hemorrhages secondary to shaken baby syndrome B Pain response over the wrists secondary to passive range of motion C Pain response with passive range of motion to the hip secondary to slipped epiphysis D Pharyngitis and sand paper rash secondary to a staph infection E Swelling and warmth over the knee secondary to hemarthroses

Swelling and warmth over the knee secondary to hemarthroses This patient has hemophilia A. Hemarthroses usually occur when an affected child begins to walk. Due to his hemophilia, easy bruising can occur. Hemarthroses can cause low-grade fevers without infection being present, so choice D is incorrect. Wrist joints are less involved then knees, ankles, and elbows.

A 33-year-old IV drug user presents to the emergency department with chills, diaphoresis, anorexia, and malaise. On physical exam, her temperature is 40C, BP 98/55, P 115 bpm, and RR 22. Two separate blood cultures are positive for S.aureus. Which of the following diagnostic studies would be most useful in establishing this patient's diagnosis? A EKG B CXR C Rheumatoid factor D ESR E TEE

TEE E Choice E, TEE (or transesophageal echocardiogram), would be most useful in establishing a diagnosis of infective endocarditis, as a positive echocardiogram demonstrating presence of a vegetation would satisfy one of the Duke criteria's major criteria. TEE is more sensitive than TTE (transthoracic echocardiogram) for detecting vegetations. EKG and CXR should be performed as part of this patient's evaluation, but would be less useful than TEE in establishing a diagnosis of infective endocarditis. ESR and rheumatoid factor are frequently elevated in patients with endocarditis, but are not specific to the diagnosis of endocarditis.

A 55-year-old right-hand dominant man presents with a 4-hour history of weakness and tingling of his right hand and numbness of the right side of his mouth. Mild difficulty was noted with word finding. His symptoms have improved since onset but have not fully resolved. There is no significant medical history. Physical examination revealed flat right nasolabial fold, subjective numbness of the right hand, right pronator drift, clumsiness of finger tapping on the right hand, increased deep tendon reflexes on the right, as well as a present Babinski. What is the most likely etiology for this patient's problem? A migraine headache B peripheral neuropathy C syncope D transient ischemic attack E seizure

TIA D Three key features of a transient ischemic attack include sudden onset and complete reversal of symptoms within 24 hours, usually within 15 minutes. The symptoms are usually in the anatomical distribution of a single blood vessel. This patient's history is not suggestive of migraine or syncope. His physical examination findings do not correlate with peripheral neuropathy or seizure.

A 24-year-old HIV-positive man comes to the emergency department complaining of severe left-sided chest discomfort, which radiates through to the left trapezius region. On coming into the room, you note that he is sitting up and hunched forward. On physical examination, the patient's blood pressure is 135/78, with a pulse of 85 bpm, and a pericardial friction rub is noted. Laboratory findings demonstrate elevated serum creatine kinase levels and normal serial troponin levels. His EKG demonstrates peaked T waves. His CXR demonstrates a "water bottle" cardiac silhouette. Which of the following diagnostic studies would be considered the most appropriate next step in management of this patient? A Cardiac catheterization B Transthoracic echocardiography C CT of the thorax D VQ scan

TTE B Choice B, transthoracic echocardiography, would allow for monitoring of a patient with acute pericarditis, to determine if a pericardial effusion and/or cardiac tamponade develops. Choice A, cardiac catheterization, would be appropriate in a patient with acute myocardial infarction. Choice C, CT of the thorax, is not as effective or specific as transthoracic echocardiography for the development of a pericardial effusion and/or tamponade. Choices D and E would be appropriate diagnostic studies if pulmonary embolism is suspected, but not as the next step in management of this patient with pericarditis

A 55-year-old woman with a history of hypertension and 2 vessel CABG presents to the emergency department with increasing dyspnea while walking up one flight of stairs. She denies chest pain and discomfort, but states that for the last 2 weeks she has also noticed palpitations. On physical examination, her vital signs are stable, with a normal physical exam. On EKG, she demonstrates atrial flutter with 2:1 AV block. Which of the following is the most appropriate next diagnostic study for this patient? A Transthoracic echocardiogram B Cardiac catheterization C Nuclear stress test D Holter monitor E Event recorder

TTE Choice A, transthoracic echocardiogram, is the most appropriate next diagnostic study in this patient with atrial flutter, as it can demonstrate the presence of valvular heart disease. The presence of valvular heart disease can change the recommendations for embolism prophylaxis. Atrial flutter is treated similarly to atrial fibrillation in terms of embolism prophylaxis. Choice B, cardiac catheterization, is useful in patients suspected to have unstable angina, or who have sustained a myocardial infarction. Choice C, nuclear stress test, is useful in patients suspected to have angina pectoris, and may be a useful diagnostic study in this patient with cardiac risk factors once the issue of atrial fibrillation has been treated. Choices D and E would be useful tests if the EKG had not established a diagnosis for this patient, with the Holter monitor indicated in patients experiencing symptoms on a daily basis, and the event recorder indicated in patients demonstrating more sporadic symptoms

A 59-year-old woman presents to the accident and emergency department by ambulance with second- and third-degree burns to her head and neck, and the anterior surfaces of her upper extremities, right leg, and trunk including her genital area. Question Which of the following represents a reasonable estimation of the extent of her burns? Answer Choices 1 36% 2 37% 3 46% 4 45% 5 55%

The correct answer is 55%. This estimation is based on the "rule of 9s". Body surface area is estimated at 9% for each arm, the head and neck, anterior surface of upper torso, anterior surface of lower torso, posterior surface of upper torso, posterior surface of lower torso, anterior surfaces of each leg, posterior surfaces of each leg and an additional 1% for the groin area for a total of 100%. In this case, 9% for her head and neck, 9% for the anterior surface of each arm, 9% for the anterior surface of her right leg, 9% for her anterior upper torso, 9% for her anterior lower torso, and 1% for the genital area for a total of 55%. The other answers are incorrect using the estimation by the "rule of 9s".

The most commonly fractured long bone in both adults and children is which of the following? A Femur B Fibula C Humerus D Radius E Tibia

Tibia The tibia is the most commonly fractured long bone in the body for both adults and children. The fractures are often the result of sporting activities in the young and may occur from a simple fall in the elderly - especially those with osteoporosis. Motor vehicle accidents are another common cause of tibial fractures. Open or complex tibial fractures are sometimes associated with compartment syndromes, infection and neurovascular compromise. The femur is the strongest of the long bones and generally only sustains fractures when exposed to extreme stress, such as that experienced in a motor vehicle collision or industrial accident. Fibular fractures commonly occur with a direct below to the lateral lower leg or with extreme ankle rotational forces or excessive inversion. A high percentage of ankle fractures involve the fibula, especially in older women. Humerus fractures are relatively rare in adults, but are the second most common fractures to occur at birth - behind only the clavicle in frequency. The radius is the most commonly fractured bone in the upper extremity, but still less common in frequency than the tibia. Falls on an outstretched arm are a common mechanism for the injury.

A 46-year-old male is evaluated for a fall from approximately 15 feet, landing on his legs. There is intense pain and swelling to the lower extremity, with decreased sensation. What bone is fractured and is most commonly associated with acute compartment syndrome? A Femur B Patella C Navicular D Talus E Tibia

Tibia he tibia is the most common bone fracture that leads to acute compartment syndrome. It can also occur after other long bone fractures of the arms and legs. While it does occur in the hands and feet, it is unlikely that the fracture of a small carpal bone (scaphoid) or tarsal bone (talus) would result in compartment syndrome. Damage to the hyoid bone in the neck and the patella in the knee are not associated with compartment syndrome.

Active elbow extension is primarily controlled by which muscle(s)? A Anconeus B Biceps C Brachialis D Brachioradialis E Triceps

Triceps E The triceps are the primary muscles that produce active elbow extension. The anconeus muscle is known as a secondary elbow extensor. The biceps and brachialis muscles are primary muscles of elbow flexion and the brachioradialis is a secondary elbow flexor.

Which type(s) of Salter-Harris fractures can generally be treated with closed reduction and cast immobilization? A Type I B Types I and II C Types I, II, and III

Type 1, 2, and 3 Minimally displaced Salter-Harris types I, II, and III fractures generally can be treated with immobilization only. Types IV and V involve the cartilage of both the articular surface and the growth plate. To ensure proper alignment and a congruous joint surfaces open reduction and internal fixation is usually necessary.

A 46-year-old female presents to the emergency department with a complaint of hematemesis, nausea, and epigastric pain. The hematemesis started earlier this morning, while the epigastric pain started three days prior and was a dull ache. She has vomited three times. She denies fever, diarrhea, or weight loss. She has not had blood in her stool. Her current medications include ibuprofen. She has been taking this medication daily for the past three months for right knee pain. What is the test that is the most sensitive for diagnosing this disease process? A Upper endoscopy B Hemoccult test C Esophageal manometry D Barium esophagography E Urea breath test

Upper endoscopy A The correct answer is upper endoscopy. The most likely diagnosis is gastritis, and upper endoscopy is the most sensitive method of diagnosis. Hemoccult testing is used to detect the presence of blood in the patient's stool. Esophageal monometry is used most often in establishing an etiology of dysphagia. Barium esophagography is also used frequently in the evaluation of patients with dysphagia. Neither esophageal manometry or barium esophagography would help you determine the source of this GI bleed. Urea breath test is used in detecting H. pylori infection, but would not be the best choice in diagnosing gastritis.

What is the initial treatment for a patient who is having an acute episode of supraventricular tachycardia? A Caffeine B Beta blockers C Valsava maneuver D No treatment E Synchronized cardioversion

Valsalva maneuver The initial treatment that should be attempted is a simple vagal maneuver to break the reentry tachycardia. If this is unsuccessful, pharmacologic therapy is indicated.

Which of the following is the most reliable clinical tool for confirming endotracheal intubation in an emergency situation? A Auscultation over the stomach B Endotracheal tube condensation C Pulse oximetry monitoring D Sellick maneuver E Visualizing the tube passing through the vocal cords

Visualizing the tube passing through the vocal cords Clinical assessments and practices used to assess tube placement, and help with placement, such as auscultating for breath sounds and noise within the stomach, have not had a confirmation rate comparable to directly visualizing the tube passing through the vocal cords. Tube condensation may occur with esophageal intubation as well. The Sellick maneuver may help with correct positioning, but is not a confirmatory test. Once placement is suspected, confirmation with an end-tidal CO 2 detector and chest x-ray is recommended. Pulse oximetry measurement should be performed throughout the intubation, with decreased saturations representing a worsening clinical condition and/or esophageal intubation.

A 52-year-old male with chronic alcoholism is brought to the emergency department by his family, due to his acting differently for several days. A physical exam reveals nystagmus, eye muscle weakness, global confusion, retrograde amnesia, and ataxia. Which of the following is the most likely etiology of the diagnosis? A Cerebrovascular accident B Hypoxemia C Uremia D Vitamin B12 deficiency E Vitamin B1 (thiamine) deficiency

Vit b1 deficiency E This patient exhibits classic symptoms associated with Wernicke encephalopathy. Wernicke encephalopathy is due to vitamin B1 (thiamine) deficiency. In the United States, this condition occurs most frequently in chronic alcoholics, but it may be seen in any condition affecting thiamine levels. Each of the other etiologies listed may also cause neurologic findings, and should be considered in the differential diagnosis and evaluation of this patient.

A 55-year-old woman with a history of mitral stenosis (secondary to rheumatic heart disease) presents to the emergency department with increasing dyspnea while walking up one flight of stairs. She denies chest pain and discomfort, but states that for last 24 hours she has also noticed palpitations. She also admits to lower extremity edema, which is new within the last week. On EKG, she demonstrates atrial flutter with 2:1 AV block. Which of the following is the most appropriate therapy for this patient? A Aspirin 325 mg B Plavix 75 mg C Plavix 75 mg and aspirin 81 mg D Warfarin, dosed to INRs between 2.0 and 3.0 E Dipyridamole 200 mg and aspirin 25 mg

Warfarin, dosed to INRs between 2.0-3.0 Patients with atrial flutter of less than 48 hour duration may be cardioverted without anticoagulation, unless they have mitral valve disease, in which case they should be treated with warfarin; therefore, choice D is the most appropriate choice. Choices A, B, C and E are inappropriate, as there is no data to support that any of these therapies offer adequate anticoagulation for embolism prophylaxis, for either atrial fibrillation or atrial flutter. Choice A is recommended for cardiac risk factor modification for men, according to the Framingham Risk Trial. Choice C is appropriate therapy following PTCA and stent placement.

The term "nursemaid's elbow" refers to which of the following physical conditions? A A fracture of the humerus B A subluxation of the radial head C Inflammation at the lateral epicondyle D Inflammation of the medial epicondyle E Olecranon bursitis

a subluxation of the radial head B This is the most common elbow injury in children under the age of 5. The injury generally occurs when the child's arm is forcefully pulled when the elbow is in an extended position and the forearm is pronated. Fibers of the annular ligament that encircle the radial neck become trapped between the radius and ulna. On presentation, children hold their arm in slight flexion and pronation. There is no fracture associated with nursemaid's elbow and the olecranon bursa is unaffected. Nursemaid's elbow is an acute injury, while medial and lateral epicondylitis (also known as golfer's elbow and tennis elbow respectively) are generally chronic conditions

A 22-year-old patient with sickle cell disease presents to the emergency department complaining of chest pain, fever, and non-productive cough. On physical exam his temperature is 100.6˚F, BP is 144/88, pulse is 110, respiratory rate is 24, and pulse oximetry is 84%. CBC shows a WBC of 11,500, hemoglobin of 8.3%, and hematocrit of 28%. What is the most likely diagnosis? A Acute bronchitis B Acute chest syndrome C Asthma D Bronchiectasis E Pneumothorax

acute chest syndrome Patients with sickle cell disease are prone to acute chest syndrome, resulting from sickling of cells within the lung that typically presents with chest pain, tachypnea, cough, fever, and oxygen desaturation. Acute bronchitis (A) and bronchiectasis (D) typically present with a productive cough without significant oxygen desaturation and anemia. Patients with asthma (C) and spontaneous pneumothorax (E) will be afebrile and have normal hemoglobin and hematocrit.


Kaugnay na mga set ng pag-aaral

Chapter 7 - Business-to-Business Marketing

View Set

Ch. 5: Chronic Illness & Older Adults

View Set